Download as pdf or txt
Download as pdf or txt
You are on page 1of 79

#91, Ground Floor, 60 Ft Main Road, Chandra Layout, Bengaluru-560040

Contact: +91-9902404443/9902504443
Website: www.india4ias.com

KADANA KAS TEST SERIES Detailed Synopsis: Test 1: Polity


1

S
1. Which of the following gave the idea of Constituent Assembly for India?
a) Simon Commission

A
b) Rajaji Formula
c) Cabinet Mission Plan
d) Wavell Plan

Solution: C
4I
IA
Explanation:
 Cabinet Mission Plan gave the idea of Constituent Assembly for India. In 1946,
British Prime Minister Clement Attlee formulated a Cabinet Mission to India.
Therefore option (c) is correct answer. Therefore option (c) is correct answer.
D

 ‘Rajaji’ formula was formulated by C. Rajagopalchari to solve the political dead-lock


between All India Muslim League and Indian-National Congress.
IN

 Wavell plan (1945) was talks between these two political parties.
 The purpose of the Simon Commission was to check the government administration
in India. In November 1927, the British government declared the application of the
Indian Statutory Commission. The aim of this commission was to examine the
Government of India act of 1919.

2. Match List-I with List-II and select the correct answer using the codes given below:
List-I List-II
(Constituent Assembly Committee) (Chairman)
A. Steering Committee 1. Sardar Vallabhbai Patel
B. Fundamental Rights Sub-Committee 2. Dr. Rajendra Prasad
C. Union Constitution Committee 3. J.B. Kripalani
D. Provincial Constitution Committee 4. Jawaharlal Nehru

Codes:
A B C D
a) 2 3 4 1
b) 1 4 3 2
c) 2 4 3 1
d) 1 3 4 2

Solution: A
Explanation:

S
Steering Committee – Rajendra Prasad

A
Fundamental Rights Sub-Committee – J.B. Kripalani
Union Constitution Committee – Jawahar lal Nehru.
4I
Provincial Constitution Committee – Sardar Vallabhbai Patel Therefore option (a) is correct
answer.
IA
3. ‘The Draft Constitution as framed only provides a machinery for the government of the
country. It is not a contrivance to install any particular party in power as has been done
in some countries. Who should be in power is left to be determined by the people, as it
must be, if the system is to satisfy the test of democracy’.
D

The above passage from Constituent Assembly debates is attributed to


a) Pandit Jawaharlal Nehru
IN

b) Dr B R Ambedkar
c) Maulana Abdul Kalam Azad
d) Acharya J B Kriplani

Solution: B
Explanation:
 On 29 August 1947, the Drafting Committee was appointed with Dr B. R. Ambedkar as
the Chairman along with six other members assisted by a constitutional advisor.
 These members were Pandit Govind Ballabh Pant, K M Munshi, Alladi Krishnaswamy
Iyer, N Gopala swami Ayengar, B L Mitter and Md. Saadullah. A Draft Constitution
was prepared by the committee and submitted to the Assembly on 4 November
1947.This is Ambedkar’s second argument rested on the legitimacy of the democratic
system. Therefore option (b) is correct answer.

4. The Preamble of the Indian constitution adopted on 26th November 1949 did not include
the terms :
1. Socialist
2. Secular
3. Integrity
4. Republic
Select the correct answer from the codes given below :
a) 1, 2 and 3

S
b) 2, 3 and 4
c) 1, 2 and 4

A
d) 3 and 4

Solution: A
Explanation:
4I
 The ideals behind the Preamble to India’s Constitution were laid down by Jawaharlal
IA
Nehru’s Objectives Resolution, adopted by the Constituent Assembly on January 22,
1947.
 Although not enforceable in court, the Preamble states the objectives of the
Constitution, and acts as an aid during the interpretation of Articles when language is
D

found ambiguous.
Components of Preamble:
IN

 It is indicated by the Preamble that the source of authority of the Constitution lies with
the people of India.
 Preamble declares India to be a sovereign, socialist, secular and democratic republic.
 The objectives stated by the Preamble are to secure justice, liberty, equality to all
citizens and promote fraternity to maintain unity and integrity of the nation.
 The date is mentioned in the preamble when it was adopted i.e. November 26, 1949.
 The term ‘Socialist’, ‘Secular’, and ‘Integrity’ were added to the preamble
through 42nd Amendment Act, 1976. Therefore option (a) is correct answer.

5. Who among the following has the authority to readjust the allocation of seats in the Lok
Sabha to the states and division of each State into territorial constituencies?
a) The President of India
b) The Parliament of India
c) The Chief Election Commissioner of India
d) Supreme court of India

Solution: B
Explanation:
According to article 82 of Indian Constitution, the parliament of India shall readjust the
allocation of seats in the Lok Sabha to the states and division of each state into territorial
constituencies. Therefore option (b) is correct answer.

S
6. Consider the following statements and select the correct answer from the codes given
below:

A
Assertion (A) : K.C. Wheare calls Indian Constitution a quasi federal Constitution.
Reason (R): There are three lists (Union, State and Concurrent) in the Seventh Schedule of the
4I
Constitution of India dividing powers between the Centre and the state and giving residuary
powers to the central government.
Codes :
IA
a) Both A and R are individually true and R is the correct explanation of A
b) Both A and R are individually true but R is not the correct explanation of A
c) A is true but R is false
d) A is false but R is true
D

Solution: A
IN

Explanation:
A federal system has a written constitution, the presence of the constitutional division
of power amongst different federal units and an independent judiciary. Quasi federalism
implies a system having a federal government with a unitary spirit. India and Canada have
quasi-federalism. Therefore option (a) is correct answer

7. Which among the following features of a federal system is not found in the Indian
Political System ?
a) Authority of the Courts to interpret the Constitution
b) Distribution of powers between the Federal and the State Governments
c) Supremacy of the Constitution
d) Dual citizenship

Solution: D
Explanation:
Following are the features of a federal system:
 Single Constitution: In true federal systems like America, Switzerland, the central and
state governments have their own separate constitutions. But in India the central and
state governments have the same constitution. Jammu-Kashmir has its own separate
constitution as it has a special constitutional status.
 Single Citizenship: Dual-citizenship can be found in countries with a model federal
system such as the United States of America, Switzerland. In such nations a person

S
holds the citizenship of both the state and nation of his birth. But single citizenship in
India is based on property. In India a person does not have the citizenship of the state

A
of his birth, but only the national citizenship, which is a clear violation of the principle
of federalism. So option D is correct. Therefore option (d) is correct answer

4I
Independent Judiciary: In a federal system, there is a need for an independent
judiciary to settle disputes that may arise between the central and state governments
regarding jurisdiction. The Supreme Court in India exercises this responsibility
IA
 Separation of Powers: Another feature of the Indian political system is the separation
of powers. In India, power is constitutionally divided between the central and state
governments.
o Union List : The Central List consists of 100 subjects of national importance
D

like Foreign Affairs, Defence, Inter-State Trade, Railways, Posts and Wires,
Income Tax, Finance, Elections, Civil Aviation, Import-Export etc. Only the
IN

Central Government has the power to make laws in respect of the matters in this
list.
o State List : The State List includes 61 subjects of local importance like Public
Health, Police, Jails, Public Peacekeeping, Commercial Taxes etc. Only the
State Government has the power to make laws in respect of the matters in this
list.
o Concurrent List : Concurrent list covers 52 subjects like Marriage, Divorce,
Education, Print Media etc. Both the Central and State Governments have the
power to make laws regarding these matters. In case of conflict between the
Acts framed by the Central and State Governments regarding these matters, the
Act of the Central Government shall prevail over the Act of the State
Government.

8. Which one among the following is not a characteristic of a federal system?


a) There are two distinct levels of government
b) The responsibilities and powers of each level of government are clearly defined in a
written Constitution
c) There is no separation of powers between the legislative and executive branches of
government
d) A Supreme Court is entrusted with the responsibility of interpreting these provisions
and arbitrating in matters of dispute

S
Solution: C

A
Explanation:
 Dual Government System: As in other countries with federal system, India also has
4I
both central and state governments. This is known as bi-government system. The
Central Government not only enacts laws relating to matters of national importance but
also administers the administration of the entire country. Each union unit has its own
IA
government which is the state government which not only makes laws relating to
matters of local importance but also administers the state administration.
 One of the features of the federal system is the distribution of power between the
federal government and the government of the states.
D

 Some powers are given to the federal government and some other remaining powers
are given to the government of states. The federal government is powerful over the
IN

matter of national (or) general importance.


 Both federal and state governments functioning strictly within the sphere
demarcated for them. Therefore option (c) is correct answer.

9. Under which article the parliament provides financial assistance to states?


a) Article 273
b) Article 274
c) Article 276
d) Article 275

Solution: D
Explanation:
 Under the Article 275 the Parliament provides financial assistance to states. This is
a central sector scheme and grants are provided to the states on the basis of ST
population percentage in the state to the total tribal population of the country.
Therefore option (d) is correct answer.
 Article 273 deals with grants in lieu of export duty on jute and jute products.
 Article 274 mentions prior recomendation of President required to Bills affecting
taxation in which states are interested.
 Article 276 is about taxes on professions, trades, callings and employments.

10. In the Indian Constitution, the right to equality is granted by which of following Articles?

S
a) Article 16 to Article 20
b) Article 14 to Article 18

A
c) Article 15 to Article 19
d) Article 13 to Article 17

Solution: B
4I
Explanation:
IA
 Article 14- Equality before Law
 Article 15- Prohibition of Discrimination
 Article 16- Equality of Opportunity
 Article 17- Abolition of untouchability
D

 Article 18- Abolition of titles. Therefore option (b) is correct answer.


 Article 19(1) in The Constitution Of India 1949, All citizens shall have the right
IN

o to freedom of speech and expression;


o to assemble peaceably and without arms;
o to form associations or unions;
o to move freely throughout the territory of India;
o to reside and settle in any part of the territory of India; and
o to practise any profession, or to carry on any occupation, trade or busin

11. Which of the following is not true of Article 32 of the Indian Constitution ?
a) It gives the Supreme Court and the High Courts the power to issue writs for the
enforcement of Fundamental Rights.
b) It is included in Part III of the Indian Constitution and is therefore itself a Fundamental
Right.
c) Dr. Ambedkar called it the ‘Heart and soul of the Indian Constitution’.
d) An aggrieved person has no right to complain under Article 32 where a Fundamental
Right has not been violated.

Solution: D
Explanation:
 Under Article 226, a High Court can issue these writs not only for the purpose of
enforcement of the fundamental rights but also for the redress of any other injury or
illegality, owing to contravention of the ordinary law. Therefore option (d) is correct

S
answer.
Important Information:

A
Habeas Corpus:
 “Habeas Corpus” means “to have the body.”

4I
This writ is used to protect an individual’s liberty and ensure that they are not
unlawfully detained.
 When issued, it commands the authorities to produce the detained person before the
IA
court to determine the legality of their detention.
 It is a remedy against illegal confinement.
 This writ can be filed by the detained person himself or his relatives or friends on his
behalf.
D

Mandamus:
 “Mandamus” means “command.”
IN

 This writ is issued by a higher court to a lower court, tribunal, or public authority,
directing them to perform a specific duty that falls within their legal jurisdiction.
 It is used to ensure that public officials and authorities perform their duties properly.
Prohibition
 “Prohibition” means “to forbid.”
 This writ is issued by a higher court to a lower court, prohibiting it from proceeding
with a case where it lacks jurisdiction or is acting beyond its powers.
 It prevents inferior courts from exceeding their authority.
 It is issued to direct inactivity and thus differs from mandamus which directs activity.
Certiorari
 “Certiorari” means “to be certified.”
 This writ is issued by a higher court to a lower court, tribunal, or authority, quashing or
transferring a case to the higher court for review.
 It is used to correct errors of jurisdiction or procedural irregularities.
 It is a curative writ.
Quo Warranto
 “Quo Warranto” means “by what authority.”
 This writ is issued to question the legal authority or right of a person to hold a public
office or position.
 It is used to prevent unauthorized or unqualified individuals from holding public office.

12. Match List -I with List - II and select the correct answer using the codes given below the

S
lists:
List – I List – II

A
(Provision) (Article number the Constitution)
A. Equality before Laws 1. Article 42
B. Right of Work
4I
C. Just and Humane conditions of of work
2. Article 45
3. Article 14
D. Free and compulsory education for children 4. Article 41
IA
Codes:
A B C D
a) 1 2 3 4
b) 3 4 1 2
D

c) 2 1 4 3
d) 4 3 1 2
IN

Solution: B
Explanation:
 Article 42 seeks to ensure humane work conditions and provide for maternity relief. It
states that “The State shall make provision for securing just and humane conditions of
work and for maternity relief”.
 Article 45 talks about the provision for free and compulsory education for children. It
states that “The State shall endeavour to provide, within a period of ten years from the
commencement of this Constitution, for free and compulsory education for all children
until they complete the age of fourteen years”.
 Article 41 of the Indian Constitution directs the state to secure the right to work,
education and public assistance in certain cases such as unemployment, old age,
sickness and disablement. It is one of the Directive Principles of State Policy based on
Socialist principles.
 Article 14 of the Constitution of India reads as under: “The State shall not deny to any
person equality before the law or the equal protection of the laws within the territory of
India.” Therefore option (b) is correct answer.

13. Consider the following statements and select the correct answer from the codes given
below :
Assertion (A) : Under Article 368, the Parliament can amend any part of the Constitution.

S
Reason (R) : The Parliament is the Supreme Legislative body elected by the people of India.
Codes :

A
a) Both (A) and (R) are correct and (R) is the correct explanation of (A)
b) Both (A) and (R) are correct, but (R) is not the correct explanation of (A)
4I
c) (A) is true, but (R) is false
d) (A) is false, but (R) is true
IA
Solution: A
Explanation:
Article 368 of the Constitution deals with the powers of Parliament to amend the
Constitution and its procedure. The Parliament of India is the supreme legislative body of the
D

Republic of India. The Indian Parliament consists of the President, Lok Sabha (the Lower
House), and the Rajya Sabha (Upper House). Therefore option (a) is correct answer.
IN

Methods of Amendments:
Our constitution can be amended through the following three methods.
1. Amendment by simple majority.
2. Amendment by Special Majority.
3. Amendment with the approval of half of the state legislatures with special majority.

1. Amendment by Simple Majority: Most parts of the Constitution can be amended by


simple majority of Parliament. Important among them are:
 Addition of new states
 Creation of new states, change of boundary or boundaries of states, change of names of
states
 Creation or Abolition of State Legislative Councils
 Salaries and allowances of President, Governor, Speaker and Judges
 Salary, Allowances and Perquisites of Members of Parliament
 Use of official language
 Quorum of Parliament (minimum number of members of the Legislative Assembly to
be present)
 Acquisition and loss of citizenship
 Elections to Parliament and State Legislative Assemblies
 Redistribution of constituencies

S
 Union Territories
 Administration of Tribal Areas

A
It should be specially noted that this type of amendment procedure by simple majority does not
come under the purview of Article 368.
4I
2. Amendment by Special Majority : Certain parts of the Constitution can be amended by
special majority of Parliament. If more than 50 percent of the total members of each House of
Parliament and 2/3 of the members present and voting approve the amendment bill, it is called
IA
a special majority. The following articles of the Constitution can be amended through this
method.
 Fundamental Rights, State Director Principles etc.
3 Amendment by special majority and ratification by half of the states : Some parts of the
D

constitution can be amended with the ratification by half of the state legislatures with special
majority of the Parliament. This rule is followed to amend matters relating to the Union system
IN

of India. Such things are,


 Method of Election of the President
 Extent of Central and State Executive Powers
 Constitution and Powers of Supreme Court and High Courts
 Division of Powers between the Center and the States
 Representation of States in Parliament
 Constitutional amendment procedure

14. Consider the following statements and select the correct answer from the codes given
below :
Assertion(a) : The All-India Services violate the federal principle of the Constitution as well
as the autonomy of States.
Reason (R) : The All-India Service officers are governed by Central Government rules and the
State Government do not have full control over them.
Codes :
a) Both A and R are individually true and R is the correct explanation of A
b) Both A and R are individually true but R is not the correct explanation of A
c) A is true but R is false
d) A is false but R is true

Solution: A

S
Explanation:
Though the all India Services violate the principle of federalism under the constitution by

A
restricting the autonomy and patronage of the states. They are supported on the ground that -
(i) they help in maintaining high standard of administration in the centre as well as in the
states.
4I
(ii) they help to ensure uniformity of the administrative system throughout the country; and
(iii) They facilitate liaison, cooperation, coordination and joint action on the issue of common
IA
interest between the Centre and States. Therefore option (a) is correct answer.

15. Consider the following statements with reference to the linguistic reorganisation of states
in independent India:
D

1. The separate state of Andhra for the Telugu people came into existence in 1953.
2. Jawaharlal Nehru was particularly in favour of the movement for linguistic reorganisation of
IN

states that came in the wake of the success of the Andhra movement.
Which one of the statements given above is/are correct?
a) Only 1
b) Only 2
c) Both 1 and 2
d) Neither 1 nor 2

Solution: A
Explanation:
 On 1 October 1953, 11 districts in the Telugu-speaking portion of Madras State became
the new Andhra State with Kurnool as the capital. Tanguturi Prakasam Pantulu (also
known as Andhra Kesari – "The Lion of Andhra") became the first Chief Minister of
the new state. Hence statement 1 is correct.
 Linguistic Provinces committee consisted of Jawahar Lal Nehru, Sardar Vallahbhai
Patel and Pattabhi Sitaramayya, submitted its report in April 1949 and formally
rejected language as the basis for reorganization of states. Hence statement 2 is
incorrect. Therefore option (a) is correct answer.
 In 1953, Jawaharlal Nehru appointed the State Reorganisation Commission. The
retired Chief Justice of the Supreme Court, Fazal Ali, was appointed as its
Chairman; its other two members were H. N. Kunzru and K. M. Panikkar.

16. In which of the following years, the Citizenship Act, 1955 has been amended?

S
1. 1986
2. 1992

A
3. 2003
4. 2005
5. 2015
6. 2019
4I
In how many years has the above been amended?
IA
a) Only two
b) Only three
c) Only four
d) All Six
D

Solution: D
IN

Explanation:
 The Citizenship Act, 1955 was amended 6 times in 1986, 1992, 2003, 2005, 2015
and 2019.
 The purpose of the Citizenship Amendment Act, 2019 is to give Indian citizenship to
illegal migrants of 6 communities i.e. Hindu, Sikh, Buddhist, Christian, Parsi, and Jain
belonging to Bangladesh, Pakistan, and Afghanistan. Therefore option (d) is correct
answer.

17. Which of the following are matters on which a constitutional amendment is possible only
with the ratification of the legislature of not less than one half of the states?
1. Election of the president
2. Representation of the states in parliament
3. Any of the lists in the 7th schedule
4. Abolition of the Legislative Council of a State.
How many of the above statements is/are correct?
a) Only one
b) Only two
c) Only three
d) All four

Solution: C
Explanation:

S
Some parts of the constitution can be amended with the ratification by half of the
state legislatures with special majority of the Parliament. This rule is followed to amend

A
matters relating to the Union system of India. Such things are,
 Method of Election of the President


4I
Extent of Central and State Executive Powers
Constitution and Powers of Supreme Court and High Courts
 Division of Powers between the Center and the States
IA
 Representation of States in Parliament
 Constitutional amendment procedure
Constitution or abolition of State Legislative Councils shall be done by amendment by
simple majority. Therefore option (c) is correct answer.
D

18. According to the Constitution of India, which one of the following rights cannot be taken
IN

away during emergency?


a) Freedom of Speech and Expression
b) Right to life
c) Right to freedom of movement
d) Right to organize

Solution: B
Explanation:
 The 6 fundamental rights (including right to freedom of speech and expression,
right to freedom, right to association) guaranteed under Article 19 of Article 358
are automatically abrogated when a state national emergency is declared.
 There is no need to issue a separate order to abrogate them, the State is free from all
restrictions imposed under Article 19 while the Emergency is in force.
 This means that the State can curtail or suspend the fundamental rights guaranteed
under Article 19 either by legislative acts or by executive decisions.
 Such laws or executive actions cannot be challenged in court on the ground that they
are inconsistent with the fundamental rights found under Article 19.
 Article 19 is revived and comes into force after the end of national emergency. Even
after the end of the emergency, there is no redress for any act of the state during the
period of emergency, i.e. the actions taken by the legislature and the executive during
the emergency cannot be questioned in any court even after the period of emergency.
 But right to life (Article - 21) cannot be taken away in case of emergency.

S
Therefore option (b) is correct answer.

A
19. According to the Indian Constitution, which one is not included in the fundamental right
to equality?
4I
a) Equality before law
b) Social equality
c) Equality of opportunity
IA
d) Economic equality

Solution: D
Explanation:
D

 Equality Before Law and Equal Protection of Laws (Article 14): Article 14 of the
Constitution makes it clear that a State shall not deny equality before law or equal
IN

protection of laws to any person within the territory of India. This right is given to
foreigners as well as citizens. Here the word person includes all legal persons i.e.
statutory corporations, companies, registered societies.
 Prohibition of Discrimination on Certain Grounds (Article 15) - Social Equality :
“Article 15(1) of the Constitution makes it clear that the State shall not discriminate
solely on the basis of religion, race, caste, sex or place of birth. The word only as used
here should be specially noted. This means that discrimination on other grounds is not
prohibited, so discrimination on grounds other than religion, race, caste, sex or place of
birth may be permitted. According to Article 15(2) a citizen shall not be subjected to
any disability, restriction or provision in respect of the following matters solely on the
ground of religion, race, caste, sex, place of birth or any of these.
 Equal opportunity in public employment (Article 16): Article 16(1) of the
Constitution states that all citizens shall be given equal opportunity in public
employment (under the State). Article 16(23) makes it clear that no discrimination shall
be made against a citizen on the basis of religion, race, caste, sex, place of birth or
domicile or any of these in the appointment to public posts under the State.
 Note: Economic equality is not mentioned in Fundamental Rights. Therefore
option (d) is correct answer.

20. Which of the following is not a fundamental duty under the Indian constitution?
a) To protect monuments of national importance
b) To develop scientific temper

S
c) To uphold the unity and integrity of the nation
d) None of these

A
Solution: A
Explanation:
4I
Note: Protection of monuments of national importance is not a fundamental duty, It is
IA
comes under DPSP Article 49. Therefore option (a) is correct answer.

11 Fundamental Duties:
 To abide by the Constitution and respect its ideals and institutions, the National Flag
D

and the National Anthem,


 To cherish and follow the noble ideals that inspired the national struggle for freedom,
IN

 To uphold and protect the sovereignty, unity and integrity of India,


 To defend the country and render national service when called upon to do so,
 To promote harmony and the spirit of common brotherhood amongst all the people of
India transcending religious, linguistic and regional or sectional diversities and to
renounce practices derogatory to the dignity of women,
 To value and preserve the rich heritage of the country’s composite culture,
 To protect and improve the natural environment including forests, lakes, rivers and
wildlife and to have compassion for living creatures,
 To develop scientific temper, humanism and the spirit of inquiry and reform,
 To safeguard public property and to abjure violence,
 To strive towards excellence in all spheres of individual and collective activity so that
the nation constantly rises to higher levels of endeavour and achievement, and
 To provide opportunities for education to his child or ward between the age of six and
fourteen years (added by the 86th Constitutional Amendment Act, 2002).
Important Insights:
 The idea of Fundamental Duties is inspired from the Constitution of Russia (erstwhile
Soviet Union).
 These were incorporated in Part IV-A of the Constitution by the 42nd Constitutional
Amendment Act, 1976 on the recommendations of the Swaran Singh Committee.
 Originally 10 in number, one more duty was added through the 86th Constitutional
Amendment Act, 2002.

S
 All the eleven duties are listed in Article 51-A of the Constitution (the sole Article in
Part-IV-A).

A
 Like the Directive Principles of State Policy, Fundamental duties are also non-
justiciable in nature.
4I
21. Match List-I with List-II and select the correct answer using the codes given below:
List-I (Writs) List-II (Ground)
IA
A. Habeas Corpus 1. Non-performance of public duties
B. Mandamus 2. Unlawful detention
C. Quo-Warranto 3. Correctional directions to subordinate courts
D. Certiorari 4. Unlawful occupation of public office
D

A B C D
a) 3 4 1 2
IN

b) 2 1 4 3
c) 3 1 4 2
d) 2 4 1 3
Solution: B
Explanation:
The Writ Jurisdiction of Supreme Court can be invoked under Article 32 of the
Constitution for the violation of fundamental rights guaranteed under Part – III of the
Constitution.
S
A
4I
22. Consider the following statements and select the correct answer from the codes given
below :
Assertion (A) : Preventive Detention is included in the chapter on Fundamental Rights in the
IA
Constitution of India.
Reason (R) : The Constitution has vested the legislative power with regard to preventive
detention in the Parliament only and the State Legislatures have no authority in this regard.
Codes:
D

a) Both A and R are individually true and R is the correct explanation of A


b) Both A and R are individually true but R is not the correct explanation of A
IN

c) A is true but R is false


d) A is false but R is true

Solution: C
Explanation:
 Preventive detention, the practice of incarcerating accused individuals before trial on
the assumption that their release would not be in the best interest of society—
specifically, that they would be likely to commit additional crimes if they were
released.
 While Parliament is exclusively entitled to enact preventive detention laws related to
defence, foreign affairs, and the security of India (List I, entry 9), both Parliament and
the state legislatures can make such laws for reasons to do with the maintenance of
public order and ensuring supplies and services essential to the community and the
security of a state (List III, entry 3). Therefore option (c) is correct answer.

23. Consider the following statements and select the correct answer from the codes given
below:
Assertion (A): Education is the fundamental right of every child between the age of 6 and 14
years.
Reason (R): The 84th Constitutional Amendment Act provided for right to education.
Codes:
a) Both A and R are individually true and R is the correct explanation of A

S
b) Both A and R are individually true but R is not the correct explanation of A
c) A is true but R is false

A
d) A is false but R is true

Solution: C
Explanation:
4I
 Originally Part IV of Indian Constitution, Article 45 and Article 39 (f) of DPSP, had a
IA
provision for state funded as well as equitable and accessible education.
 The first official document on the Right to Education was Ramamurti Committee
Report in 1990.
 In 1993, the Supreme Court’s landmark judgment in the Unnikrishnan JP vs State of
D

Andhra Pradesh & Others held that Education is a Fundamental right flowing from
Article 21.
IN

 Tapas Majumdar Committee (1999) was set up, which encompassed insertion of
Article 21A.
 The 86th amendment to the constitution of India in 2002, provided Right to Education
as a fundamental right in part-III of the Constitution.
 The same amendment inserted Article 21A which made Right to Education a
fundamental right for children between 6-14 years.
 The 86th amendment provided for a follow-up legislation for Right to Education Bill
2008 and finally Right to Education Act 2009. Hence statement 1 is correct and 2 is
incorrect. Therefore option (c) is correct answer.

Features of Right to Education (RTE) Act, 2009:


 The RTE Act aims to provide primary education to all children aged 6 to 14 years.
 It enforces Education as a Fundamental Right (Article 21).
 The act mandates 25% reservation for disadvantaged sections of the society where
disadvantaged groups include:
o SCs and STs
o Socially Backward Class
o Differently abled

24. Which one of the following rights conferred by the Constitution of India is also available
to non-citizens?
a) Freedom to acquire property or to carry on any occupation, trade or business.

S
b) Freedom to move, reside and settle in any part of the territory of India
c) Freedom of speech, assembly and form association

A
d) Right to constitutional remedies

Solution: A
Explanation:
4I
IA
Fundamental rights available only to Fundamental rights available to both
citizens and not to foreigners citizens & foreigners (except enemy aliens)
D

Article 15: Prohibition of discrimination on


Article 14: Equality before law and equal
grounds of religion, race, caste, sex or place
protection of laws.
IN

of birth.

Article 16: Equality of opportunity in matters Article 20: Protection in respect of


of public employment. conviction for offences.

Article 19: Protection of six rights


regarding freedom of (speech and Article 21: Protection of life and personal
expression, (ii) assembly (iii) association, liberty.
(iv) movement (v) residence and profession.
Article 29: Protection of language, script and
Article 21A: Right to elementary education.
culture of minorities.

Article 30: Right of minorities to establish Article 22: Protection against arrest and
and administer educational institutions. detention in certain cases.

Article 23: Prohibition of traffic and human


beings and forced labour.

S
Article 24: Prohibition of employment of
children in factories etc.

A
4I Article 25: Freedom of conscience and free
profession, practice and propagation of
religion.
IA
Article 26: Freedom to manage religious
affairs.
D

Article 27: Freedom from payment of taxes


IN

for promotion of any religion.

Article 28: Freedom from attending religious


instruction or worship in certain educational
institutions.

Note: Freedom to acquire property or to carry on any occupation, trade or business: This right
is guaranteed under Article 19(1)(g) of the Constitution of India. This right is available not
only to citizens but also to non-citizens, including foreigners. Therefore option (a) is correct
answer.
25. With reference to the Constitution of India, which one of the following pairs is not
matched?
a) Forests : Concurrent List
b) Stock Exchanges : Concurrent list
c) Post office saving Bank : Union List
d) Public Health : State list

Solution: B
Explanation:
Stock exchange and future markets are listed in the Union list, but not in the

S
concurrent list. Hence option (b) is correct answer.
Important Insights:

A
 Article 246 of the Constitution mentions three lists in the Seventh Schedule — union,
state and concurrent lists. While the Centre can make laws on subjects specified in the


4I
Union list, the state governments have jurisdiction over items in the state list.
Both the Centre and states can make laws for subjects in the concurrent list, but the
Union's law will prevail in case of conflict.
IA
26. Sarkaria Commission was set up to review -
a) the relation between the President and the Prime Minister
b) the relation between the legislative and the executive
D

c) the relations between the executive and the judiciary


d) the relations between the Union and the State.
IN

Solution: D
Explanation:
The Sarkaria Commission, formally known as the "Commission on Centre-State
Relations," was a commission established by the Government of India. It was set up to
examine and recommend changes to the distribution of powers and responsibilities between
the central government and state governments. Hence option (d) is correct answer.
Important Insights:
 Administrative reforms commission recommendations:
 Establishment of an Inter-state council under Article 263 of the constitution.
 Appointment of persons having long-experiences in public life and non-partisan
attitudes as governors.
Punchhi commission:
 Giving a fixed term of five years to the governors and their removal by the process of
impeachment
 Union should be extremely restrained in asserting Parliamentary supremacy in matters
assigned to the states

27. Which of the following statements with regard to Inter-State Council is/are correct?
1. It was established under the provisions of the Constitution of India.
2. The Council is a recommendatory body.

S
3. There is a standing committee of the Council under the Chairmanship of the Prime
Minister of India to process matters for consideration of the Council.

A
Select the correct answer using the codes given below
a) Only 1 and 3
b) Only 2
4I
c) Only 1 and 2
d) All of the above
IA
Solution: C
Explanation:
 The Inter-State Council was established under Article 263 of the Constitution of
D

India through a Presidential Order dated 28 May 1990. Hence statement 1 is


correct.
IN

 As the article 263 makes it clear that the Inter-State Council is not a permanent
constitutional body for coordination between the States of the Union.
 It can be established ‘at any time’ if it appears to the President that the public interests
would be served by the establishment of such a Council.
 The Council is a recommendatory body. The Council shall consist of Prime Minister
(Chairman), Chief Ministers of all States and union territories (Member),
Administrators (UT) and Six Ministers of Cabinet rank to be nominated by the Prime
Minister (Member).This is not a standing committee. Hence statement 2 is correct
and 3 is incorrect. Therefore option(c) is correct answer.

28. In what way does the Indian Parliament exercise control over the administration?
a) Through Parliamentary Committee
b) Through Consultative Committee of various ministers
c) By making the administration send periodic reports
d) By compelling the executives to issue writs

Solution: A
Explanation:
 Parliament supervises the Activities of the executives with the help of its
committees like committees on Govt. assurance, committees on subordinate
legislation, committees on petition etc. Therefore option (a) is correct answer.
 Considering the volume of legislation to be handled by the Parliament, committees

S
have been established to transact a good deal of the legislation.
 Parliamentary Committees are of two kinds - Standing Committees and Ad hoc

A
Committees.
 The former are elected or appointed periodically and they work on a continuous basis.
4I
The latter are appointed on an ad hoc basis as need arises and they cease to exist as
soon as they complete the task assigned to them.
IA
29. Consider the following statements:
1. An amendment of the Constitution of India can be initiated by the introduction of a Bill only
in the Lok Sabha.
2. The Bill for amendment of the Constitution of India has to be passed in each House by a
D

majority of the total membership of that House and by a majority of not less than two-thirds of
the members of that House Present and voting.
IN

Which of the statements given above is / are correct?


a) Only 1
b) Only 2
c) Both 1 and 2
d) Neither 1 nor 2

Solution: B
Explanation:
Procedure for amendment of constitution:
 Amendment can be initiated only by the introduction of a bill for the purpose
in either house of the Parliament. Hence statement 1 is incorrect.
 The bill can be introduced either by a minister or by a private member and
does not require the permission of the President
 The bill must be passed in each house by a special majority, that is, a majority of
the total membership of the house and a majority of two-thirds of the members of the
house present and voting. Hence statement 2 is correct. Therefore option (b) is
correct answer.
 Each house must pass the bill separately. If there is any disagreement, there is no
provision for joint sitting of the houses
 The president must give his assent to the bill. He can neither withhold his assent to
the bill nor return the bill for reconsideration of the Parliament.

S
30. Which of the following statements regarding Supreme Court of India are correct?
1. It has the power to entertain appeal from any court or tribunal within India.

A
2. It has the power to deliver advisory opinion on any question of fact or law referred to it by
the President.
4I
3. Salaries of the Judges of the Supreme Court are subject to vote by the Parliament.
4. Its jurisdiction is binding on all other courts within India.
How many of the above statements is/are correct?
IA
a) Only one
b) Only two
c) Only three
d) All four
D

Solution: C
IN

Explanation:
The Supreme Court is the highest Court appeal from all courts in India. It hears appeals in –
(i) Cases involving interpretation of the constitution- civil, Criminal or otherwise (Article 132)
(ii) Civil cases irrespective of any constitutional issue (Article. 133)
(iii) Criminal matters irrespective of any constitutional issue (134).
Besides, the Supreme Court may grant special leave to appeal in certain cases (136).
(Statement 1,2 and 4 are correct). Therefore option (c) is correct answer.
Note: Salaries of the Judges of the Supreme Court are subject to not vote by the
Parliament.

31. Which one of the following statements about the Parliament of India is not correct?
a) The Constitution provides for a parliamentary form of government.
b) The foremost function of the parliament is to provide a cabinet.
c) The membership of the cabinet is restricted to the lower house.
d) The cabinet has to enjoy the confidence of the majority in the popular chamber.

Solution: C
Explanation:
 Article 74 (COM to aid and advice President): The advice tendered by Ministers to the
President shall not be inquired into in any court.
 The President may require the COM to reconsider such advice and the President shall
act in accordance with the advice tendered after such reconsideration.

S
 Article 75 (Other Provisions as to Ministers): The PM shall be appointed by the
President and the other Ministers shall be appointed by the President on the advice of

A
the PM.
 The total number of ministers, including the Prime Minister, in the COM shall not
4I
exceed 15% of the total strength of the Lok Sabha. This provision was added by the
91st Amendment Act of 2003.
 A minister who is not a member of the Parliament (either house) for any period of
IA
six consecutive months shall cease to be a minister. Therefore option (c) is correct
answer.

32. The first speaker against whom a motion of no-confidence was moved in the Lok Sabha
D

was
a) Balram Jakhar
IN

b) G.V. Mavalankar
c) Hukum Singh
d) K.S. Hegde

Solution: B
Explanation:
 The first speaker against whom a motion of non-confidence was moved in the Lok
Sabha was G.V. Mavalankar. Ganesh Vasudev Mavalankar (27 November 1888 –
27 February 1956) popularly known as Dadasaheb term – (1952 – 1956).
Therefore option (b) is correct answer.
 Balram Jakhar - 8th Speaker of Lok Sabha.
 Hukum singh - 3rd Speaker of Lok Sabha.
 K. S. Hegde - 7th Speaker of Lok Sabha.

33. In terms of Parliamentary terminology, What do we call a rule of legislative procedure


under which a further debate on a motion can be stopped?
a) Closure
b) Guillotine
c) Expunction
d) Recess

Solution: A

S
Explanation:
A rule of legislative procedure under which further debate on a motion can be stopped is

A
known in parliamentary terminology as Closure. Hence option (a) is correct answer.
Important Information:

4I
The Parliamentary Recess is a break in the parliamentary session during which neither
house meets to undertake business.
 "Expunction"-Deletion of words, phrases or expressions from the proceedings or
IA
records of Rajya Sabha by an order of the Chairman for being defamatory or indecent
or unparliamentary or undignified.
 Often it happens that all the clauses of a bill are not get discussed within the allotted
D

time, as a result, the undiscussed clauses of the bill or resolution are also put for voting
along with the discussed clauses for saving time. This process is called Guillotine
Closure.
IN

34. When an ordinary Bill is referred to a joint sitting of both the Houses of Indian
Parliament, it has to be passed by a
a) simple majority of the total number of members of both the Houses present and voting.
b) two-third majority of the total number of members of both the Houses.
c) simple majority of the total number of members of both the Houses.
d) two-third majority of the total number of members of both the Houses present and
voting.

Solution: A
Explanation:
Joint session is presided over by the speaker of Lok Sabha or in his absence by the Deputy
Speaker. In a joint sitting a bill is passed simply by the majority of the both the members of the
house present and voting. Hence option (a) is correct answer.

Important Information:
Article 108: Article 108 of the Indian Constitution of India provides for Joint sitting of both
the Houses. The joint sitting of the Parliament is called by the President and is presided over
by the Speaker or, in his absence, by the Deputy Speaker of the Lok Sabha or in his absence,
the Deputy-Chairman of the Rajya Sabha.

35. Which one of the following statements is correct?

S
a) There is a constitutional provision for nominating two members belonging to the
Anglo-Indian Community to the Rajya Sabha.

A
b) There is no constitutional bar for nominated members to be appointed as a Union
Minister.
4I
c) A nominated member can vote both in the Presidential and Vice Presidential elections.
d) None of the above
IA
Solution: B
Explanation:
There is no bar on appointment of a person from outside the legislature as Minister, but
he cannot continue as Minister for more than 6 months unless he secures a seat in either house
D

of the parliament (by election or nomination) in the meantime [Article 75(5)]. Hence option
(b) is correct answer.
IN

Important Information:
 In January 2020, the Anglo-Indians were abolished by the 104th Constitutional
Amendment Act, 2019, which earlier reserved seats in the Parliament and State
Legislatures of India.
 The presidential electoral college is made up of the following: elected members of the
Rajya Sabha (upper house of the Parliament of India); elected members of the Lok
Sabha (lower house of the Parliament of India); elected members of each state's
Legislative Assembly (lower house of the state legislature);
 The Vice-President is elected by an electoral college consisting of members of both
Houses of Parliament, in accordance with the system of proportional representation by
means of the single transferable vote and the voting in such election is by secret ballot.
36. Consider the following statements and select the correct answer from the codes given
below:
Assertion (A): The President of India can return any bill passed by Parliament for
reconsideration of the Houses.
Reason (R): The President cannot return money bills to the Parliament for reconsideration of
the Houses.
Codes:
a) Both A and R are individually true and R is the correct explanation of A
b) Both A and R are individually true but R is not the correct explanation of A
c) A is true but R is false

S
d) A is false but R is true

A
Solution: D
Explanation:
4I
All bills passed by the Parliament can become laws only after receiving the assent of the
President. After a bill is presented to him, the President shall declare either that he assents to
the Bill, or that he withholds his assent from it. As a third option, he can return a bill to the
IA
Parliament, if it is not a money bill or a constitutional amendment bill, for reconsideration.
When, after reconsideration, the bill is passed and presented to the President, with or without
amendments, the President cannot withhold his assent from it. Hence option (d) is correct
answer.
D

Important Information:
Ordinary Bills:
IN

 Ordinary bill is introduced in any House of Parliament. This bill is introduced by a


Minister or a Private Member of Parliament.
 An ordinary bill is concerned with any issue other than financial concerns, according
to Articles 107 and 108 of the Indian Constitution.
 There is no recommendation of the President in the case of ordinary bills.
 Ordinary bills can be changed or rejected by Rajya Sabha, and they can be delayed for
six months.
 Article 111 of the Indian Constitution requires that a bill be brought to the President
for his approval or assent after it has been enacted by both houses of Parliament.
 In the event of an ordinary bill, there is a provision for a joint sitting.
Money Bill:
 Article 110 (3) of the Constitution, “If any question arises whether a Bill is a Money
Bill or not, the decision of the Speaker of the House of the People thereon shall be
final.
 Money Bills must be introduced in the Lok Sabha and cannot be introduced in the
Rajya Sabha (the upper house).
 The Rajya Sabha can only make recommendations on a Money Bill but does not have
the power to amend or reject it.
 President can either accept or reject a money bill but cannot return it for
reconsideration.
 There is no provision for Joint sitting.

S
37. Which of the following is/are correct in respect of ‘Zero Hour’ discussion?

A
1. It is not directed against individual Minister.
2. It covers questions raised over matters of public importance.
Select the correct answer using the codes given below:
a) Only 1
b) Only 2
4I
c) Both 1 and 2
IA
d) Neither 1 nor 2

Solution: B
D

Explanation:
 During the Zero Hour members raise matters of importance, especially those which
cannot be delayed (. It can directed against individual Minister). Nobody knows which
IN

issue a member would raise during this hour. As a result, questions so raised without
prior notice result in avoidable loss of precious time of the House. Hence option (b) is
correct answer.
 Zero Hour is the first hour of every sitting in both houses of Parliament. It is mentioned
in the Rules of Business of the houses of Parliament.

38. A Member of Lok Sabha does not become disqualified to continue as a Member of the
House if the Member:
a) Voluntarily gives up his / her membership of the political party from which he /she was
elected.
b) Is expelled by the political party from which he / she had been elected to the House.
c) Joins a political party after being elected as an independent candidate.
d) Abstains from voting contrary to the direction by his / her political party.

Solution: B
Explanation:
An MP elected from a particular party does not become disqualified for a member of a house,
when he is expelled by that particular party. Hence option (b) is correct answer.
ImportantInformation:
Anti Defection Law:
 The anti-defection law punishes individual Members of Parliament (MPs)/Members of
the Legislative Assembly (MLAs) for leaving one party for another.

S
 Parliament added it to the Constitution as the Tenth Schedule in 1985 in order to bring
stability to governments by discouraging legislators from changing parties.

A
 The Tenth Schedule - popularly known as the Anti-Defection Act - was included in the
Constitution via the 52nd Amendment Act, 1985.


4I
Grounds for Defection:
Voluntary Give Up: If an elected member voluntarily gives up his membership of a
political party.
IA
 Violation of Instructions: If he votes or abstains from voting in such House contrary to
any direction issued by his political party or anyone authorized to do so, without
obtaining prior permission.
D

 Elected Member: If any independently elected member joins any political party.
 Nominated Member: If any nominated member joins any political party after the expiry
of six months.
IN

The decision on questions as to disqualification on ground of defection are referred to the


Chairman or the Speaker of such House, which is subject to ‘Judicial review’.

39. Consider the following statements relating to the procedure of the election of the Speaker
and the Deputy Speaker of the Lok Sabha
1. The election of a Speaker shall be held on such date as the Prime Minister may fix and the
Secretary General shall send to every member notice of this date.
2. The election of a Deputy Speaker shall be held on such date as the Speaker may fix and the
Secretary General shall send to every member notice of this date.
3. At anytime before noon on the day preceding the date so fixed, any member may give notice
in writing of a motion that another member be chosen as the Deputy Speaker of the House.
How many of the statement(s) given above is/are correct?
a) Only one
b) Only two
c) All three
d) None

Solution: B
Explanation:
Election of Speaker shall be held on such date as the President may fix, and the
Secretary-General shall send notice of this dateto every member. The election of a Deputy
Speaker shall be held on such date as the Speaker may fix and the Secretary-General shall

S
send notice of this dateto every member. Hence statement 1 is incorrect and 2 is correct.
At any time before noon on the day preceding the date so fixed, any member may

A
give notice in writing, addressed to the Secretary-General, of a motion that another
member be chosen as the Deputy Speaker of the House and the notice shall be seconded by a
4I
third member and shall be accompanied by a statement by the member whose name is
proposed in the notice that he is willing to serve as Deputy Speaker, if elected. Hence
statement 3 is correct. Therefore option (b) is correct anwer.
IA
40. Which one of the following statements regarding the Departmental Committee of the
Parliament of India on the empowerment of women is correct?
a) The Committee will consist of members of the Lok Sabha only
D

b) A Cabinet Minister can be a member of the Committee.


c) The term of office of the members of the Committee shall not exceed two years
IN

d) It reports on the working of welfare programmes for the women.

Solution: D
Explanation:
 The Committee on Empowerment of Women was constituted on 29th April 1997.
 The Committee consists of 30 Members of whom 20 are nominated by the Speaker
from amongst the Members of Lok Sabha and 10 are nominated by the Chairman,
Rajya Sabha from amongst the Members of the Rajya Sabha.
 A Minister cannot be nominated a Member of the Committee.
 The term of the Committee does not exceed one year.
 One of the functions of the committee is to report on the working of the welfare
programmes for the women. Therefore option (d) is correct anwer.

41. Which of the following statements with respect to the judiciary in India is/are correct?
1. Under the Constitution of India, there is a single integrated system of courts for the Union as
well as the states.
2. The organisation of the subordinate judiciary varies slightly from state to state.
Select the correct answer using the codes given below
a) Only 1
b) Only 2

S
c) Both 1 and 2
d) None of the above

A
Solution: C
Explanation:

4I
One of the unique features of the Indian Constitution is that, notwithstanding the
adoption of a federal system and existence of Central Acts and State Acts in their
IA
respective spheres, it has generally provided for a single integrated system of
Courts to administer both Union and State laws.
 At the apex of the entire judicial system, exists the Supreme Court of India below
D

which are the High Courts in each State or group of States. Below the High Courts,
lies a hierarchy of Subordinate Courts. Therefore option (c) is correct anwer.
IN

42. Who among the following Indian Prime Ministers resigned before facing a vote of no-
confidence in the Lok Sabha?
a) Chandra Shekhar
b) Morarji Desai
c) Chaudhary Charan Singh
d) V.P. Singh

Solution: C
Explanation:
 Chaudhary Charan Singh was the Prime Minister who resigned before facing a vote of
no-confidence in the Lok Sabha. Chaudhuri Charan Singh was the sixth Prime
Minister, serving from 28 July 1979 until 14 January 1980. Therefore option (c) is
correct answer.

43. Which one of the following is part of the Electoral College for the election of the
president of India but does not form part of the forum for his impeachment?
a) Lok Sabha
b) Rajya Sabha
c) State Legislative Councils
d) State legislative Assemblies

Solution: D

S
Explanation:
There is no direct election for the Indian President. An electoral college elects him.

A
President Related Constitutional Provisions:
 Article 54: Election of President


4I
Article 55: Manner of election of President.
Article 56 :Term of office of President
 Article 57: Eligibility for re-election.
IA
 Article 58: Qualifications for election as President
The electoral college responsible for President’s elections comprises elected members of:
 Lok Sabha and Rajya Sabha
D

 Legislative Assemblies of the states (Legislative Councils have no role) (but not
participate in impingement process). Therefore option (d) is correct answer.
 Legislative Assemblies of the Union Territories of Delhi and Puducherry.
IN

The following group of people is not involved in electing the President of India:
 Nominated Members of Rajya Sabha (12)
 Nominated Members of State Legislative Assemblies
 Members of Legislative Councils (Both elected and nominated) in bicameral
legislatures
 Nominated Members of union territories of Delhi and Puducherry
President Impingement Process:
S
A
44. Department of Border management comes under of which one of the following Ministry?
4I
a) Ministry of Defense
b) Ministry of Home Affairs.
c) Ministry of Shipping, Road Transport and Highways
IA
d) Ministry of Environment and Forest.

Solution: B
Explanation:
D

Department of border management is a department of ministry of Home affairs of the


union minister. Department of Border Management is dealing with management of borders,
IN

including coastal borders. Therefore option (b) is correct answer.

45. In which year Supreme Court of India came into being?


a) 1937 January 28
b) 1947 January 28
c) 1950 January 28
d) 1949 January 28

Solution: C
Explanation:
On the 28th of January, 1950, two days after India became a Sovereign Democratic
Republic, the Supreme Court came into being. The inauguration took place in the Chamber of
Princes in the Parliament building which also housed India’s Parliament, consisting of the
Council of States and the House of the People. Therefore option (c) is correct answer.

46. Which of the following Constitution Amendment Acts seeks that the size of the Councils
of Ministers at the Centre and in State must not exceed 15 per cent of the total number of
members in the Lok Sabha and the total numbers of members of the Legislative
Assembly of that state, respectively?
a) 91st

S
b) 93rd
c) 95th

A
d) 97th

Solution: A
Explanation:
4I
 The Constitution (91st Amendment) Act, 2003 inserted clause 1A in Article 164,
IA
which says “the total number of Ministers, including the Chief Minister, in the Council
of Ministers in a State shall not exceed 15% of the total number of members of the
Legislative Assembly of that State. Therefore option (a) is correct answer.

D

It also Inserted clause 1A in Article 75, the total number of ministers, including the
Prime Minister, in the COM shall not exceed 15% of the total strength of the Lok
Sabha.
IN

 The purpose of the 91st Amendment was to prevent jumbo Cabinets and the resultant
drain on the public exchequer.
Imporatnt Insights:
The 93rd Amendment Act gives the State permission to set up reservation policies in private,
non-public educational institutions.
97th Amendment: The amendment introduced the right to form cooperative societies as a
fundamental right under Article 19 of the Constitution. It also introduced a new Directive
Principle of State Policy, as per Article 43B, emphasizing the state's role in promoting
cooperative societies.
47. The power to enlarge the jurisdiction of the Supreme Court of India with respect to any
matter included in the Union list of legislative powers rests with:
a) The President of India
b) The Chief Justice of India
c) The Parliament
d) The Union Ministry of Law

Solution: C
Explanation:
Apart from the jurisdictions enumerated in the Constitution, the jurisdiction of the
Supreme Court can be widened by the Parliament by making law in this regard. Article 138 of

S
the Constitution deals with the power of the Parliament for granting extra jurisdiction to
the Supreme Court by making law. Therefore option (c) is correct answer.

A
48. Article 136 of Indian Constitution authorizes the Supreme Court to grant special leave to
4I
appeal. Which of the following statement is not correct with respect to ‘Appeal by Special
Leave’?
a) It is a discretionary power of the Supreme Court.
IA
b) It can be granted against any court or tribunal including the military court.
c) It can be related to any matter
d) None of these
D

Solution: B
Explanation:
IN

Special Leave Petition:


Under Article 136 of the constitution, the supreme court is authorised to grant in its discretion
special leave to appeal from any judgement in any matter passed by any Court or Tribunal in
the country ( except military Tribunal and court martial). This provision contains the four
aspects as under;
 It is a discretionary power and cannot be granted as a matter of right
 It can be granted in any judgement whether final or interlocutory
 It can be related to any matter whether constitutional, revenue, civil etc.
 It can be granted against any court or tribunal and not necessarily against High
Court (except military Tribunal and court martial). Therefore option (b) is
correct answer.
49. The Legislative Council in a State in India can be created or abolished by the -
a) Parliament on the recommendation of a Governor of the state.
b) Parliament alone can establish or abolish
c) Parliament after the state assembly passes the resolution of that effect.
d) Governor of the state on the recommendation of the Council of Ministers.

Solution: C
Explanation:
Article 169 (Creation and Abolition):
The Parliament can abolish a Legislative Council (where it already exists) or create it

S
(where it does not exist) by a simple majority, that is, a majority of the members of each
House present and voting, if the legislative assembly of the concerned state, by a special

A
majority, passes a resolution to that effect.
Special majority implies


4I
A majority of the total membership of the assembly and
A majority of not less than two-thirds of the members of the assembly present and
voting. Therefore option (c) is correct answer.
IA
Important Information:
Legislative Council:
Six States having a Legislative Council: Andhra Pradesh, Telangana, Uttar Pradesh, Bihar,
D

Maharashtra, Karnataka.
Composition:
 Under Article 171 of the Constitution, the Legislative Council of a state shall not have
IN

more than one-third of the total strength of the State Assembly, and not less than 40
members.
 Like the Rajya Sabha, the legislative council is a continuing chamber, that is, it is a
permanent body and is not subject to dissolution. The tenure of a Member of the
Legislative Council (MLC) is six years, with one-third of the members retiring every
two years.
Manner of Election:
 One-third of the MLCs are elected by the state’s MLAs,
 Another 1/3rd by a special electorate comprising sitting members of local governments
such as municipalities and district boards,
 1/12th by an electorate of teachers and another 1/12th by registered graduates.
 The remaining members are appointed by the Governor for distinguished services in
various fields namely, literature, science, art, cooperative movement and social service.

50. Article 156 of the constitution of India provides that a Governor shall hold office for a
term of five years from the date on which he enters upon his office. Which of the
following can be deduced from this?
1. No Governor can be removed from his office till the completion of this term.
2. No Governor can continue in office beyond a period of five years.
a) 1 only
b) 2 only
c) Both 1 and 2

S
d) Neither 1 nor 2

A
Solution: D
Explanation:
4I
Article 156 of the Indian Constitution deals with the Term of office of Governor. It
further states the following: The Governor shall hold office during the pleasure of the
President. The Governor may, by writing under his hand addressed to the President,
IA
resign his office.

The term of governor's office is normally 5 years but it can be terminated earlier by:

 Dismissal by the president on the advice of the council of minister headed by the prime
D

minister of the country.

 Dismissal of governors without a valid reason is not permitted.


IN

 However, it is the duty of the President to dismiss a governor whose acts are upheld by
courts as unconstitutional and malafide. Hence both the given statements are
incorrect. Therefore option (d) is correct answer.

Important Information:

 The oath of office to the governor is administered by the Chief Justice of the concerned
High Court

Constitutional Provisions related to Governor:


 The appointment and powers of government can be derived from Part VI of the Indian
constitution. Article 153 says that there shall be a Governor for each State. One person
can be appointed as Governor for two or more States.

 The governor acts in 'Dual Capacity' as the Constitutional head of the state and as the
representative.

 He is the part of federal system of Indian polity and acts as a bridge between union and
state governments.

Immunity:

 He enjoys personal immunity from legal liability for his official acts

S
 During his term of office, he is immune from any criminal proceedings, even in respect

A
of his personal acts. He cannot be arrested or imprisoned

 However, after giving two months’ notice civil proceedings can be instituted against
him during his term of office in respect of his personal acts.
4I
51. Consider the following statements.
IA
1. In 1977, the Janata Party government appointed a Commission of Inquiry headed by Justice
J.C.Shah.
2. It was appointed to inquire into several aspects of allegations of abuse of authority, excesses
and malpractices committed and action taken in the wake of the Emergency proclaimed in
D

1975.
Which of these statements are not correct?
IN

a) 1 only
b) 2 only
c) Both of them
d) Neither of them

Solution: D
Explanation:
Shah Commission of Inquiry In May 1977, the Janata Party government appointed a
Commission of Inquiry headed by Justice J.C. Shah, retired Chief Justice of the Supreme Court
of India, to inquire “into several aspects of allegations of abuse of authority, excesses and
malpractices committed and action taken in the wake of the Emergency proclaimed on the 25th
June, 1975”. Hence statement 1 is correct.
The Commission examined various kinds of evidence and called scores of witnesses to
give testimonies. These included Indira Gandhi who appeared before the Commission but
refused to answer any questions. The Government of India accepted the findings, observations
and recommendations contained in the two interim reports and third and final report of the
Shah Commission. The reports were also tabled in the two houses of Parliament. Hence
statement 2 is correct. Therefore option (d) is correct answer.

52. Parliament of India consists of which of the following:


1. President
2. The Rajya Sabha
3. The Lok Sabha

S
How many of the statement(s) given above is/are correct?

A
a) Only one
b) Only two
c) All three
d) None
4I
IA
Solution: C
Explanation:
 The Parliament of India is the supreme legislative body of the Republic of India. It is
a bicameral legislature composed of the President of India and the two houses: the
D

Rajya Sabha (Council of States) and the Lok Sabha (House of the People). Hence
option (c) is correct answer.
IN

 The President in his role as head of legislature has full powers to summon and prorogue
either house of Parliament or to dissolve Lok Sabha. The president can exercise these
powers only upon the advice of the Prime Minister and his Union Council of Ministers.
Those elected or nominated (by the President) to either house of Parliament are referred
to as Members of Parliament (MP).
 The Members of Parliament, Lok Sabha are directly elected by the Indian public voting
in Single-member districts and the Members of Parliament, Rajya Sabha are elected by
the members of all State Legislative Assembly by proportional representation. The
Parliament has a sanctioned strength of 543 in Lok Sabha and 245 in Rajya Sabha
including the 12 nominees from the expertise of different fields of science, culture, art
and history. The Parliament meets at Sansad Bhavan in New Delhi.
53. With reference the Rule of Law, consider the following statements:
1. Members of the Constituent Assembly instituted several provisions in the Constitution that
would
establish the rule of law.
2. Rule of law means that all laws apply equally to all citizens of the country and no one can be
above the law.

Which of the statements given above is/are correct?


a) 1 only
b) 2 only

S
c) Both of them
d) Neither of them

A
Solution: C
Explanation:

4I
Members of the Constituent Assembly were agreed there should be no arbitrary xercise
of power in independent India. They, therefore, they instituted several provisions in the
IA
Constitution that would establish the rule of law. The most important of these was that
all persons in independent India are equal before the law. The law cannot discriminate
between persons on the basis of their religion, caste or gender.
 What the rule of law means is that all laws apply equally to all citizens of the country
D

and no one can be above the law. Neither a government official, nor a wealthy person
nor even the President of the country is above the law. Any crime or violation of law
IN

has a specific punishment as well as a process through which the guilt of the person has
to be established. Hence both given statements are correct. Therefore option (c) is
correct answer.

54. Consider the following regarding Council of ministers:


1. Council of Ministers is the official name for the body that includes all the Ministers.
2. Cabinet is the inner ring of the Council of Ministers.
3. Ministers of State are attached to and required to assist Cabinet Ministers.
4. Parliamentary democracy in most countries is often known as the Cabinet form of
government.
How many of the statement(s) given above is/are correct?
a) Only one
b) Only two
c) Only three
d) All four

Solution: D
Explanation:
 Council of Ministers is the official name for the body that includes all the Ministers. It
usually has 60 to 80 Ministers of different ranks. Cabinet Ministers are usually top-
level leaders of the ruling party or parties who are in charge of the major ministries.
 Usually the Cabinet Ministers meet to take decisions in the name of the Council of

S
Ministers. Cabinet is thus the inner ring of the Council of Ministers. It comprises about
20 ministers. Ministers of State with independent charge are usually in-charge of

A
smaller Ministries. They participate in the Cabinet meetings only when specially
invited.

4I
Ministers of State are attached to and required to assist Cabinet Ministers. Since it is
not practical for all ministers to meet regularly and discuss everything, the decisions
are taken in Cabinet meetings. That is why parliamentary democracy in most countries
IA
is often known as the Cabinet form of government.
 The Cabinet works as a team. The ministers may have different views and opinions, but
everyone has to own up to every decision of the Cabinet. No minister can openly
criticise any decision of the government, even if it is about another Ministry or
D

Department.
 Every ministry has secretaries, who are civil servants. The secretaries provide the
IN

necessary background information to the ministers to take decisions.


 The Cabinet as a team and is assisted by the Cabinet Secretariat. This includes many
senior civil servants who try to coordinate the working of different ministries. Hence
both given statements are correct. Therefore option (d) is correct answer.

55. In the context of India, which one of the following is the characteristic appropriate for
bureaucracy?
a) An agency for widening the scope of parliamentary democracy
b) An agency for strengthening the structure of federalism
c) An agency for facilitating political stability and economic growth
d) An Agency for the implementation of public policy
Solution: D
Explanation:
Bureaucracy is the executive arm of the government. In traditional classical literature on
organs of government, one studied the legislature, the executive and the judiciary with
bureaucracy being subsumed under the executive (which comprises the political establishment
and the bureaucracy). Therefore option (d) is correct answer.

56. Which of the following advocated for universal adult franchise during pre-independence
period in India?
1. The Constitution of India Bill, 1895

S
2. Nehru report, 1928
3. Sapru Report, 1945

A
4. Government of India act, 1919
5. Lucknow pact, 1916
4I
Select the correct answer using the code given below:
a) 1, 2 and 3 only
IA
b) 1, 2 and 4 only
c) 2, 3 and 4 only
d) 3, 4 and 5 only
D

Solution: A
Explanation:
IN

A significant strand of the Indian freedom movement was centered around demands for
universal adult franchise. the Constitution of India Bill 1895, the Nehru report 1928 and Sapru
Report 1945 advocated for universal adult franchise in pre independence period in India.
Earliest of effort which advocated for universal adult franchise was the Constitution of India
Bill 1895 that gave every citizen ‘a right to give one vote for electing a member to the
Parliament of India and one local Legislative Council. Hence statement 1 is correct.

The Nehru Report 1928 stated that ‘Every person of either sex who has attained the age of 21,
and is not disqualified by law, shall be entitled to vote’. In the years leading up to the setting
up of the Constituent Assembly, more Indian historical constitutions like the Gandhian
Constitution of Free India 1946, Ambedkar's States and Minorities 1945, provided for
universal adult franchise. Hence statement 2 is correct.

The Constitutional Proposals of the Sapru Committee commonly referred to as the Sapru
Committee Report was published in 1945 to resolve issues pertaining to minorities that had
plagued Indian political and constitutional discourse. It was prepared by a committee
appointed by the Non-Party Conference in November 1944. It advocated for the Union
Assembly there shall be adult franchise, for seats other than those reserved for special
interests. Hence statement 3 is correct. Therefore option (a) is correct answer.

Important Insights:

S
 Government of India act 1919, did not provide for adult suffrage. The voting right then
was confined to certain eligibility criteria like property ownership, land ownership,

A
payment of income and municipal tax. The Government of India Act of 1919 allowed
the provincial council to decide whether women could vote provided they met stringent


4I
property, income of educational criteria.
Lucknow Pact 1916, authored jointly by the Indian National Congress and the Muslim
League, called for ‘as broad a franchise as possible’. It did not call for universal adult
IA
franchise.

57. Which of the following statements most correctly describes the term citizenship?
a) Citizenship is the freedom of a person to purchase property within a country.
D

b) Citizenship is a right which is guaranteed by birth.


c) Citizenship is a relationship between an individual and a country.
IN

d) Citizenship is the entitlement of residence of any person in a country.

Solution: C
Explanation:
Citizenship is a bond/relationship between an individual and a country to which the
individual owes allegiance and is entitled to protection in return. Citizenship implies a state of
liberty with associated responsibilities. Certain rights, obligations, and responsibilities are
granted to all citizens but are denied or only partially provided to non-citizens residing in that
country. It is also about citizencitizen relations and involves certain obligations of citizens to
each other and to the society. Equality of rights and status is one of the basic rights of
citizenship. It is not merely the freedom to purchase property within a country. Also, non-
citizen too can purchase property upon fulfilling certain conditions. Therefore option (c) is
correct answer.

58. The Act passed during the British rule made a beginning of the representative
institutions by associating Indians with the law-making process. It also provided for the
establishment of new legislative councils for Bengal, North-Western Provinces and
Punjab. It also empowered the Viceroy to issue ordinances, without the concurrence of
the legislative council, during an emergency.
The Act described in the paragraph given above was:
a) Indian Councils Act of 1909
b) Indian Councils Act of 1861

S
c) Indian Councils Act of 1892
d) Charter Act of 1853

A
Solution: B
Explanation:
4I
After the great revolt of 1857, the British Government felt the necessity of seeking the
cooperation of the Indians in the administration of their country. The Indian Councils Act of
IA
1861 is an important landmark in the constitutional and political history of India.
Therefore option (b) is correct answer.
The features of this Act were as follows:
1) It made a beginning of the representative institutions by associating Indians with the law-
D

making process. It, thus, provided that the Viceroy should nominate some Indians as non-
official members of his expanded council. In 1862, Lord Canning, the then Viceroy, nominated
IN

three Indians to his legislative council–the Raja of Benaras, the Maharaja of Patiala and Sir
Dinkar Rao.

2) It initiated the process of decentralisation by restoring the legislative powers to the Bombay
and Madras Presidencies. It, thus, reversed the centralising tendency that started from the
Regulating Act of 1773 and reached its climax under the Charter Act of 1833. This policy of
legislative devolution resulted in the grant of almost complete internal autonomy to the
provinces in 1937.

3) It also provided for the establishment of new legislative councils for Bengal, North-Western
Provinces and Punjab, which were established in 1862, 1886 and 1897, respectively.
4) It empowered the Viceroy to make rules and orders for the more convenient transaction of
business in the council. It also gave a recognition to the ‘portfolio’ system, introduced by Lord
Canning in 1859. Under this, a member of the Viceroy’s council was made in-charge of one or
more departments of the Government and was authorised to issue final orders on behalf of the
council on matters of his department(s).

5) It empowered the Viceroy to issue ordinances, without the concurrence of the legislative
council, during an emergency. The life of such an ordinance was six months.

59. Which of the following powers are enjoyed by President?

S
1. All governmental activities take place in the name of the President.
2. All laws and major policy decisions of the government are issued in President’s name

A
3. All international treaties and agreements are made in the name of the President.
4. The President is the supreme commander of the defence forces of India.
4I
How many of the statement(s) given above is/are correct?
a) Only one
b) Only two
IA
c) Only three
d) All four

Solution: D
D

Explanation:
Following are the powers of the President. But the President exercises all these powers only on
IN

the advice of the Council of Ministers.


1. All governmental activities take place in the name of the President.
2. All laws and major policy decisions of the government are issued in her name.
3. All international treaties and agreements are made in the name of the President.
4. The President is the supreme commander of the defence forces of India.
5. A bill passed by the Parliament becomes a law only after the President gives assent to it. If
the President wants, she can delay this for some time and send the bill back to the Parliament
for reconsideration. But if the Parliament passes the bill again, she has to sign it. Hence both
given statements are correct. Therefore option (d) is correct answer.

60. With reference to Lok Adalat, which of the following statements is correct?
a) The Legal Services Authorities Act, 1987 provides for the establishment of Permanent
Lok Adalat.
b) Lok Adalat deals with matters which are civil and not criminal in nature.
c) Every Lok Adalat consists of either serving or retired judicial officers only.
d) Lok Adalat have the jurisdiction to settle the matters at pre-litigative stage and not
those matters pending before any court.

Solution: A
Explanation:
 Permanent Lok Adalats are organized under Section 22-B of The Legal Services
Authorities Act, 1987. Permanent Lok Adalats have been set up as permanent bodies

S
with a Chairman and two members for providing compulsory pre-litigative mechanism
for conciliation and settlement of certain cases.

A
 These cases are relating to Public Utility Services like transport, postal, telegraph etc.
Here, even if the parties fail to reach to a settlement, the Permanent Lok Adalat gets
4I
jurisdiction to decide the dispute, provided, the dispute does not relate to any offence.
Further, the Award of the Permanent Lok Adalat is final and binding on all the parties.
The jurisdiction of the Permanent Lok Adalats is upto Rs. Ten Lakhs. Therefore
IA
option (a) is correct answer.

61. The power to increase the number of judges in the Supreme Court of India is vested in
a) the President of India
D

b) the Parliament
c) the Chief Justice of India
IN

d) the Law Commission

Solution: B
Explanation:
 The power to increase the number of judges in the Supreme Court of India is vested in
the parliament. At present, the Supreme Court consists of thirty-four judges (one chief
justice and thirty-three other judges). In 2019, the centre notified an increase in the
number of Supreme Court judges from thirty-one to thirty-four, including the Chief
Justice of India.
 This followed the enactment of the Supreme Court (Number of Judges) Amendment
Act, 2019. Originally, the strength of the Supreme Court was fixed at eight (one chief
justice and seven other judges). Therefore option (b) is correct answer.

62. Consider the following statements about Public Account Committee:


1. This committee was first setup in 1911 under the provisions of the Government of India Act
of 1909.
2. The members are elected according to the principle of proportional representation by means
of single transferable vote.
3. It is vested with the power of disallowance of expenditures by the departments.
4. Only the Parliament can take a final decision on its findings.

S
Which of the above statements is/are correct?

A
a) 1 and 3 only
b) 1, 2 and 3 only
c) 2 and 4 only
4I
d) 2, 3 and 4 only
IA
Solution: C
Explanation:
 This committee was set up first in 1921 under the provisions of the Government of
India Act of 1919 and has since been in existence It is not vested with the power of
D

disallowance of expenditures by the departments. Hence statement 1 and 3 are


incorrect.
IN

 The members are elected according to the principle of proportional representation by


means of single transferable vote. And Only the Parliament can take a final decision on
its findings. Hence statement 2 and 4 are correct. Therefore option (c) is correct
answer.

63. With reference to the Parliament of India, which of the following Parliamentary
Committees scrutinizes and reports to the House whether the powers to make
regulations, rules, sub -rules, by -laws, etc. conferred by the Constitution or delegated by
the Parliament are being properly exercised by the Executive within the scope of such
delegation?
a) Committee on Subordinate Legislation
b) Committee on Government Assurances
c) Rules Committee
d) Business Advisory Committee

Solution: A
Explanation:
The Committee on Subordinate legislation examines and reports to the House whether
the powers to make regulations, rules, sub-rules and bye-laws delegated by the Parliament or
conferred by the Constitution to the Executive are being properly exercised by it. In both the
Houses, the committee consists of 15 members. It was constituted in 1953. Therefore option
(a) is correct answer.

S
64. Consider the following statements regarding a No-Confidence Motion in India:

A
1. There is no mention of a No-Confidence Motion in the Constitution of India.
2. A motion of No-Confidence can be introduced in the Lok Sabha only.
4I
Which of the statements given above is/are correct?
a) 1 only
IA
b) 2 only
c) Both 1 and 2
d) Neither 1 nor 2
D

Solution: C
Explanation:
IN

 The principle of collective responsibility implies that the Lok Sabha can remove the
ministry (i.e., council of ministers headed by the prime minister) from office by passing
a vote of no confidence.
 There is no mention of a No-Confidence Motion in the Constitution of India. However,
Rule 198 of the Rules of Procedure and conduct of Lok Sabha specifies the procedure
for moving a no-confidence motion. Hence Statement 1 is correct.
 A Council of Ministers is collectively responsible to Lok Sabha and it remains in office
till it enjoys confidence of majority of the members in Lok Sabha. Thus, A no-
confidence motion can be moved only in the Lok Sabha (or state assembly as the case
may be). A no-confidence motion can be moved by any member of the House. Hence
Statement 2 is correct. Therefore option (c) is correct answer.
65. Rajya Sabha has equal powers with Lok Sabha in
a) the matter of creating new All India Services
b) amending the Constitution
c) the removal of the government
d) making cut motions

Solution: B
Explanation:
Rajya Sabha has equal power with Lok Sabha under article 368 of the
Constitution providing for Power of Parliament to amend the Constitution. An

S
amendment to the Constitution can be initiated only by the introduction of a Bill for the
purpose in either House of Parliament, and when the Bill is passed in each House by a majority

A
of the total membership of that House and by a majority of not less than two-thirds of the
members of that House present and voting, it is to be presented to the President who shall give
4I
his assent to the Bill and thereupon the Constitution shall stand amended. Therefore option
(b) is correct answer.
IA
66. Which of the following changes occurred in the Constituent Assembly of India after the
partition of India on August 15, 1947?
1. The number of members in the Assembly was reduced to 299.
2. Most of the Princely states choose not to join the Constituent Assembly of India.
D

3. None of the members of the Muslim League joined the Constituent Assembly of India.
IN

Select the correct answer using the code given blow:


a) 1 and 2 only
b) 2 and 3 only
c) 1 only
d) 1, 2 and 3

Solution: C
Explanation:
 After the commencement of Partition, The Muslim league members which are from the
areas included in Pakistan withdrew from the constituent assembly. So, the total
strength came down to 299 as against 389 proposed under cabinet mission plan. The
strength of provinces also reduced from 296 to 229. Hence statement 1 is correct.
 Initially they had decided to stay away from it and so did not join it but after the
acceptance of Mountbatten plan, representatives of most of the Princely states took
their seats in the constituent assembly. However, the strength of Princely states was
reduced to 70 after the partition instead of 93 as fixed under Cabinet mission. Hence
statement 2 is incorrect.
 The members of the Muslim League from the Indian Dominion entered the Constituent
Assembly of India. Twenty-eight members of the Muslim League joined the Indian
Assembly. Hence statement 3 is incorrect. Therefore option (c) is correct answer.

S
67. Which of the following Indian states have bicameral legislature?
1. Andhra Pradesh

A
2. Bihar
3. Maharashtra
4I
4. Jammu and Kashmir
5. Karnataka
IA
Select the correct answer using the code given below:
a) 1, 2, 3 and 5 only
b) 1, 3 and 4 only
c) 2, 4 and 5 only
D

d) 1, 2, 3, 4 and 5
IN

Solution: A
Explanation:
Bicameralism is the federal feature of the constitution. The Constitution provides
for a bicameral legislature consisting of an Upper House (Rajya Sabha) and a Lower House
(Lok Sabha). The Rajya Sabha represents the states of Indian Federation, while the Lok Sabha
represents the people of India as a whole. The Rajya Sabha (even though a less powerful
chamber) is required to maintain the federal equilibrium by protecting the interests of the states
against the undue interference of the Centre.
Of the 28 states and 8 Union Territories of India, only 6 states that is Andhra
Pradesh, Bihar, Karnataka, Maharashtra, Telangana and Uttar Pradesh - have bicameral
legislatures, while the rest all have unicameral legislatures. In the six states with bicameral
legislatures, the upper house is called the Legislative Council (Vidhan Parishad) one-third of
whose members are elected every two years. Hence option 1, 2, 3 and 5 only are correct.
Therefore option (a) is correct answer.

68. With reference to the Secretariat of the Parliament, consider the following statements:
1. The Secretariat of the Parliament is a statutory office.
2. The Secretary-General of the Lok Sabha secretariat is appointed by the President.

Which of the above statements is/are correct?


a) 1 only
b) 2 only

S
c) Both 1 and 2
d) Neither 1 nor 2

A
Solution: D
Explanation:

4I
The Constitution of India provides for a separate secretarial staff for each House of
Parliament. Parliament may by law regulate the recruitment and the conditions of
IA
service of persons appointed to the secretarial staff of either House of Parliament. The
Secretariat functions under the overall guidance and control of the Chairman/ Speaker.
 Article 98 of the Indian Constitution states that each House of Parliament shall have
separate secretarial staff of its own, though there can be some posts common to both
D

the Houses. Thus, it is a constitutional office and not a statutory office. Hence
statement 1 is incorrect.
IN

 The secretariat of each House is headed by a secretary-general. He is a permanent


officer and is appointed by the presiding officer of the House (and not the
President). Hence statement 2 is incorrect. Therefore option (d) is correct answer

69. With reference to Departmental standing committees, consider the following statements:
1. A standing committee consists of 21 members entirely from Lok Sabha.
2. They consider the national basic long-term policy documents presented to the Houses.
3. These committees cannot consider the matters of day-to-day administration of the concerned
ministries.

Select the correct answer using the code given below:


a) 1 and 2 only
b) 2 and 3 only
c) 3 only
d) 1, 2 and 3

Solution: B
Explanation:
 The main objective of the standing committees is to secure more accountability of the
Executive (i.e., the Council of Ministers) to the Parliament, particularly financial
accountability. They also assist the Parliament in debating the budget more effectively.
Currently there are 24 Departmental standing committees functioning in India. 17

S
Departmentally-Related Standing Committees (DRSCs) were set up in the Parliament
in 1993. In 2004, seven more such committees were setup, thus increasing their number

A
from 17 to 24.
 Each standing committee consists of 31 members in which 21 are from Lok Sabha
4I
and 10 are from Rajya Sabha. The members of the Lok Sabha are nominated by the
Speaker from amongst its own
 members, just as the members of the Rajya Sabha are nominated by the Chairman from
IA
amongst its members. Hence Statement 1 is incorrect.
 The functions of each of the standing committees are to consider the demands for
grants of the concerned ministries / departments before they are discussed and voted in
the Lok Sabha. It also considers national basic long-term policy documents
D

presented to the Houses. Hence Statement 2 is correct.


 One of the limitations of these standing committees is that they should not consider the
IN

matters of day-to-day administration of the concerned ministries / departments.


Hence Statement 3 is correct. Therefore option (b) is correct answer.

70. Which one of the following Articles of the Constitution of India was referred to as ‘dead
letter’ by Dr.B R Ambedkar?
a) Article 356
b) Article 352
c) Article 34
d) Article 360

Solution: A
Explanation:
 Since 1950, the President's Rule has been imposed on more than 100 occasions, that is,
on an average twice a year. Further, on a number of occasions, the President's Rule has
been imposed in an arbitrary manner for political or personal reasons. Hence, Article
356 has become one of the most controversial and most criticised provision of the
Constitution.
 Dr BR Ambedkar, while replying to the critics of Article 356 (President’s rule)
provision in the Constituent Assembly, hoped that the drastic power conferred by
Article 356 would remain a dead-letter'and would be used only as a measure of last
resort. He also said, if the Centre is to interfere in the administration of provincial
affairs, it must be under some obligation which the Constitution imposes upon the

S
Centre. The proper thing we ought to expect is that such Articles will never be called
into operation and that they would remain a dead-letter. Therefore option (a) is

A
correct answer.

4I
71. Which one of the following suggested that the Governor of a state should be an eminent
person from outside the State and should be a detached figure without intense political
links or should not have taken part in politics in the recent past?
IA
a) First Administrative Reforms Commission (1966)
b) Rajamannar Committee (1969)
c) Sarkaria Commission (1983)
d) National Commission to Review the Working of the Constitution (2000)
D

Solution: C
IN

Explanation:
The aforementioned was a suggestion of Sarkaria Commission. Sarkaria Commission
(1983) was a three-member commission on centre-state relations appointed by PM Indira
Gandhi led central government. It was constituted under the chairmanship of RS Sarkaria, a
retired judge of Supreme Court. Therefore option (c) is correct answer.

The important recommendations are mentioned below:


1) A permanent Inter-State Council called the Inter-Governmental Council should be set up
under Article 263.
2) Article 356 (President’s Rule) should be used very sparingly, in extreme cases as a last
resort when all the available alternatives fail.
3) The institution of All-India Services should be further strengthened and some more such
services should be created.
4) The residuary powers of taxation should continue to remain with the Parliament, while the
other residuary powers should be placed in the Concurrent List.
5) When the President withholds his assent to the state bills, the reasons should be
communicated to the state government.

72. The Governor submits reports of which of the following bodies to the State Legislature?
1. The State Finance Commission
2. The State Information Commission
3. The State Public Service Commission

S
4. The State Human Rights Commission

A
Select the correct answer using the code given below.
a) 1 and 2 only
4I
b) 1 and 3 only
c) 2, 3 and 4 only
d) 1, 3 and 4 only
IA
Solution: B
Explanation:
 Usually, Reports of Constitutional Bodies are submitted to the Governor who lays
D

those reports before the state legislature.


 Governor lays the reports of the State Finance Commission, the State Public
IN

Service Commission and the Comptroller and Auditor General relating to the accounts
of the state, before the state legislature. Hence Options 1 and 3 are correct.
 In general, reports of Statutory Bodies are submitted to State Government. For
example, the State Human Rights Commission submits its annual or special reports
to the state government. These reports are laid before the state legislature, along with a
memorandum of action taken on the recommendations of the Commission and the
reasons for non-acceptance of any of such recommendations. Hence Option 4 is
incorrect.
 Similarly, the reports of the State Information Commission are submitted an annual
report to the State Government on the implementation of the provisions of this Act.
Hence Option 2 is incorrect. Therefore option (b) is correct answer.
73. Consider the following statements:
1. The Executive Power of the Union of India is vested in the Prime Minister.
2. The Prime Minister is the ex Officio Chairman of the Civil Services Board.

Which of the statements given above is/are correct?


a) 1 only
b) 2 only
c) Both 1 and 2
d) Neither 1 nor 2

S
Solution: D
Explanation:

A
All executive actions of the Government of India are formally taken in the name of
President. He can make rules specifying the manner in which the orders and other instruments
4I
made and executed in his name shall be authenticated. He also can make rules for more
convenient transaction of business of the Union government, and for allocation of the said
business among the ministers. Thus, the executive power of the Union of India is not vested in
IA
the Prime Minister, but in the President. Hence Statement 1 is incorrect.
Cabinet secretary (not Prime Minister) is the ex-officio chairman of the Civil Services
Board. The Cabinet Secretariat functions directly under the Prime Minister. The administrative
head of the Secretariat is the Cabinet Secretary who is also the ex-officio Chairman of the
D

Civil Services Board. The business allocated to Cabinet Secretariat under Government of India
(Allocation of Business) Rules, 1961 includes (i) Secretarial assistance to the Cabinet and
IN

Cabinet Committees; and (ii) Rules of Business. Hence Statement 2 is incorrect. Therefore
option (d) is correct answer.

74. Article 164 of the Indian Constitution provides for the appointment of a Minister in
charge of Tribal Welfare. In which of the following states, does the appointment of such a
Minister has been provided by the Constitution?
1. Bihar
2. Jharkhand
3. Chhattisgarh
4. Odisha
5. Rajasthan
Choose the correct answer from the codes given below:
a) 2, 3, 4 and 5 only
b) 1, 3 and 4 only
c) 2, 3 and 4 only
d) 1, 2 and 5 only

Solution: C
Explanation:
 Article 164 deals with the appointment of a Minister in charge of tribal welfare in
certain states. The Minister may also be in charge of the welfare of Scheduled Castes
and backward classes or any other work. Presently, the Governor needs to appoint

S
such Minister in the four states. These are Jharkhand, Chhattisgarh, Madhya
Pradesh and Odisha.

A
 Originally, this provision was applicable to Bihar, Madhya Pradesh and Odisha. The
94th Amendment Act freed Bihar from this obligation because there are no
4I
Scheduled areas in Bihar now. This Amendment Act, however, to the newly formed
states of Chhattisgarh and Jharkhand. This provision has never been applicable to
Rajasthan. Therefore option (c) is correct answer.
IA
75. With reference to the Prime Minister (PM) of India, consider the following statements:
1. PM is advised by the President with regard to summoning and proroguing of the sessions of
the Parliament.
D

2. PM advises the President with regard to the appointment of members of the UPSC.
3. PM is the Chairman of the Inter State Council.
IN

Which of the statements given above are correct?


a) 2 and 3 only
b) 1 and 3 only
c) 1 and 2 only
d) 1, 2 and 3

Solution: A
Explanation:
The Prime Minister is the leader of the Lower House. In this capacity, he enjoys certain
powers. The Prime Minister advises the President with regard to summoning and
proroguing of the sessions of the Parliament. The Prime Minister can recommend
dissolution of the Lok Sabha to President at any time. The Prime Minister announces
government policies on the floor of the House. Hence Statement 1 is incorrect.

The Prime Minister advises the President with regard to the appointment of important
officials like
1) Attorney general of India,
2) Comptroller and Auditor General of India,
3) Chairman and members of the UPSC,
4) Election commissioners and
5) Chairman and members of the finance commission. Hence Statement 2 is correct.
The Prime Minister is the chairman of the NITI Aayog (which succeeded the

S
planning commission), National Integration Council, Inter State Council, National Water
Resources Council and some other bodies. Hence Statement 3 is correct. Therefore option

A
(a) is correct answer.

4I
76. Which of the following is common to the election for the membership of both Rajya
Sabha and Lok Sabha?
a) Every citizen above the age of 18 is an eligible voter.
IA
b) A voter can cast vote for multiple candidates.
c) The candidate contesting election must be registered as an elector from any
parliamentary constituency.
d) The winner must get more than half of the votes polled.
D

Solution: C
IN

Explanation:
The candidate for election to parliament must be registered as an elector for a
parliamentary constituency. This is same in the case of both, the Rajya Sabha and the Lok
Sabha. The requirement that a candidate contesting an election to the Rajya Sabha from a
particular state should be an elector in that particular state was dispensed with in 2003. In
2006, the Supreme Court upheld the constitutional validity of this change. Therefore option
(c) is correct answer.

77. Which among the following is a special feature that distinguishes a pressure group from
a political party?
a) Parties take political stances, while pressure groups do not bother about political issues.
b) Pressure groups do not seek to get into power, while political parties do.
c) Pressure groups are always confined to a few people, while parties involve larger
number of people.
d) Pressure groups do not seek to mobilise people, while political parties do.

Solution: B
Explanation:
A pressure group is a group of people who are organised actively for promoting and defending
their common interest. Pressure groups are organisations that attempt to influence
government policies. But unlike political parties, pressure groups do not aim to directly
control or share political power. Organisations with the objective of attempting to influence the

S
Government policies are known as Pressure groups. Pressure groups are the people with
common views and policies. Promoting the interests of a group of society or interests of

A
particular sections of society is the usual aim of Pressure or interest groups. Professional
associations of teachers, doctors, lawyers; business associations, trade unions are some
4I
examples of Pressure or interest groups. Therefore option (b) is correct answer.

78. The provisions of Part IX of the Constitution are not applicable to which one of the
IA
following groups of states?
a) Sikkim, Manipur and Nagaland
b) Jammu & Kashmir, Mizoram and Meghalaya
c) Assam, Jammu & Kashmir and Manipur
D

d) Nagaland, Mizoram and Meghalaya


IN

Solution: D
Explanation:
 The 73rd Constitutional Amendment Act is not applicable to Nagaland; Mizoram and
Meghalaya States, as the traditional local institutions of self-government exist in these
States.
 The government recently amended J&K Panchayati Raj Rules, 1996, to provide for
establishment of elected District Development Councils in Jammu and Kashmir. It
will comprise members directly elected from 14 territorial constituencies of the district
as well as the members of the Legislative Assembly and the chairperson of all Block
Development Councils of the district. The move to have elected third tier of
Panchayati Raj institution marks the implementation of the entire 73rd Amendment
Act in J&K. Therefore option (d) is correct answer.

79. With reference to the effects of the proclamation of National Emergency on the life of
Lok Sabha consider the following statements:
1. The life of Lok Sabha could be extended by only six months at a time.
2. The term of 5th Lok Sabha was extended than its normal period, as National emergency
was in force at that time.
3. Parliament can extend the life of Lok Sabha by passing a law to that effect with simple
majority.
How many of the statement(s) given above is/are correct?

S
a) Only one
b) Only two

A
c) All three
d) None

Solution: B
4I
Explanation:
IA
 The term of the Lok Sabha can be extended during the period of national emergency
by a law of Parliament for one year at a time for any length of time. However, this
extension cannot continue beyond a period of six months after the emergency has
ceased to operate. Hence statement 1 is incorrect.
D

 The term of the Fifth Lok Sabha which was to expire on 18 March, 1976 was extended
by one-year up to 18 March, 1977 by the House of the People (Extension of Duration)
IN

Act, 1976. It was extend for a further period of one year up to 18 March, 1978 by the
House of the People (Extension of Duration) Amendment Act, 1976. However, the
House was dissolved on 18 January, 1977, after having been in existence for a period
of five years, ten months and six days. Hence statement 2 is correct.
 During National Emergency, the duration of Lok Sabha can be extended to a maximum
of 1 year at a time by passing a law in the Parliament. The term of 5th Lok Sabha was
extended twice by-passing House of the People (extension of duration) Act, 1976.
And such a law can be passed by a simple majority. Hence statement 3 is correct.
Therefore option (b) is correct answer.
80. With reference to Special provisions for some states given in part XXI of the
Constitution, separate development boards are provided for which of the following
regions?
1. Vidarbha, Marathwada and the rest of Maharashtra
2. Saurashtra, Kutch and the rest of Gujarat
3. Hyderabad-Karnataka region
4. Naga Hills-Tuensang Area

Select the correct answer using the code given below:


a) 1, 2 and 3 only
b) 2, 3 and 4 only

S
c) 1, 2, 3 and 4
d) 1, 3 and 4 only

A
Solution: A
Explanation:

4I
Articles 371 to 371-J in Part XXI of the constitution contain special provisions for
twelve states viz., Maharashtra, Gujarat, Nagaland, Assam, Manipur, Andhra Pradesh,
IA
Telangana, Sikkim, Mizoram, Arunachal Pradesh, Goa and Karnataka. The intention
behind them is to meet the aspirations of the people of backward regions of the states
or to protect the cultural and economic interests of the tribal people of the states or to
deal with the disturbed law and order condition in some parts of the states or to protect
D

the interests of the local people of the states.


 Under Article 371, the President is authorized to provide that the Governor of
IN

Maharashtra and that of Gujarat would have special responsibility for: the
establishment of separate development boards for (1) Vidarbha, Marathwada and the
rest of Maharashtra, (2) Saurashtra, Kutch and the rest of Gujarat. Hence statements 1
and 2 are correct.
 Under Article 371-J, the President is empowered to provide that the Governor of
Karnataka would have special responsibility for the establishment of a separate
development board for Hyderabad-Karnataka region. Hence statements 3 is correct.
 Article 371-A makes special provisions for Nagaland. But it does not include the
provision for a development board. However, the Governor may specify on the
recommendation of the regional council, that the administration of the Tuensang
district shall be carried on by the Governor. Hence statement 4 is incorrect.
Therefore option (a) is correct answer.

81. Consider the following statements about World Economic Forum


1. It was established in 1971 as a not-for-profit foundation
2. It is headquartered in Geneva, Switzerland.
Which of the above statements is/are correct?
a) Only 1
b) Only 2
c) Both 1 and 2
d) Neither 1 nor 2

S
Answer: C

A
Explanation :

 It was established in 1971 as a not-for-profit foundation and is headquartered in


4I
Geneva, Switzerland.
 The World Economic Forum is the International Organization for Public-Private
sector Cooperation.
IA
 The foundation's stated mission is "improving the state of the world by engaging
business, political, academic, and other leaders of society to shape global,
regional, and industry agendas". The Forum programmatically promotes a multi-
D

stakeholder governance model, stating that the world is best managed by a self-
selected coalition of multinational corporations, governments and civil society
organizations (CSOs), which it expresses through initiatives like the "Great Reset"
IN

and the "Global Redesign".


 The 53rd edition of the annual World Economic Forum summit 2023 was held in
Davos, Switzerland. Highligths of the same are as follows :
o Climate Action in Focus: The 2023 WEF prioritized tackling the climate
crisis, with discussions on accelerating the transition to renewable energy,
scaling up carbon capture technologies, and building climate resilience in
vulnerable communities.
o Geopolitical Tensions Take Center Stage: The ongoing war in Ukraine
and its global ramifications dominated discussions, with leaders urging for
diplomatic solutions, economic stability, and humanitarian assistance.
o Cost of Living Crisis and Inequality: The rising cost of living and
widening wealth gap were pressing concerns, with calls for policies to
address inflation, support vulnerable populations, and promote inclusive
economic growth.
o Technological Innovation for a Sustainable Future: The potential of
emerging technologies like artificial intelligence, blockchain, and quantum
computing to address global challenges like climate change and healthcare
was explored.
o Collaboration for a More Resilient World: The importance of
international cooperation and multilateralism in tackling complex
challenges like pandemics, climate change, and cyber threats was

S
emphasized.

A
82. Consider the statements below:
1. The Wassenaar Arrangement is a multilateral export control regime (MECR) which
4I
controls export of conventional arms and dual-use goods and technologies.
2. The Australia Group is an informal group of countries to help member countries to
identify those exports which need to be controlled so as not to contribute to the spread
IA
of chemical and biological weapons.
Which of the above statements is/are correct?
a) 1 only
b) 2 only
D

c) Both 1 and 2
d) None of the above
IN

Answer: C

Explanation
Context : India to assume chairmanship of Wassenaar Arrangement for one year from Jan 1 st
2023.

Multilateral Export Control Regime (MECR): There are FOUR major MECR

 Wassenaar Arrangement: The Wassenaar Arrangement has been established in


order to contribute to regional and international security and stability, by promoting
transparency and greater responsibility in transfers of conventional arms and
dual-use goods and technologies, thus preventing destabilising accumulations.
India was inducted to WA on 7th Dec 2017 as the 42nd member.
 Nuclear Suppliers Group (NSG): It aims to prevent the proliferation of nuclear
weapons by controlling the export of nuclear materials and technologies. NSG
came into being as a response to the 1974 Nuclear test by India. It has 48 members.
India is not a member.
 Missile Technology Control Regime (MTCR): It aims to limit the spread of
ballistic missiles and other unmanned delivery systems that could be used for
chemical, biological, and nuclear attacks. Formed in 1987 by G-7 countries. It has
35 members. India was inducted into MTCR in 2016 as 35th member.

S
 Australia Group: It aims to prevent the spread of chemical and biological
weapons by controlling the export of related materials and technologies.

A
Formed in 1985 was prompted by Iraq’s use of chemical weapons during the Iran-
Iraq war. It has 43 members. India joined on 19th Jan 2018.
4I
83. What is the main objective of Cloud Forest 25 (CF25) initiative?
a) Investment Initiative to protect Mountain tropical Forest
IA
b) Cloud seeding to increasing forest cover to more than 33% by 2025
c) It’s a new development in Cloud seeding to increase rain forming Clouds
d) New cloud based app developed to monitor increase in forest cover
D

Answer: A
IN

Explanation:

 Recently a new report “Cloud Forest Assets Financing a Valuable Nature-Based


Solution" was released by Earth Security, a global nature-based asset management
advisory firm.
 Cloud Forests are mountain tropical forests generally found at the river
headstreams and mostly covered with clouds.
 These forests serve as the storage of clean water for communities, industries and
hydropower plants.
 Majority of Cloud Forest i.e., 90% are found in 25 developing countries in tropical
regions which bears the disproportionate impact of climate change.
 Cloud Forest 25 (CF25) Investment Initiative to establish a collective of all 25
countries that have high cloud forest to accelerate the international application of
market templates and aggregate the blended finance and data needed to achieve
solutions at scale.
 Dedicated financial bonds can help capture economic value of cloud forests in 25
countries Finance should be mobilised for cloud forest protection through payments
schemes targeting 25 developing countries, public finance institutions, private sector.

 Cloud forest bonds are debt-based instruments to mobilize finance for protection of
cloud forest. Their value is based on the economic value of a country’s Cloud forest
resources.

S
A
84. Currently which among the following country is highest contributor of troops to UN
peace Keeping Force
a) India
b) China
c) Bangladesh
4I
d) USA
IA
Answer: C

Explanation:
D

Currently Bangladesh is contributing 6,500 soldiers to United Nations peacekeeping


missions, the highest number of any country. Nepal followed with 5,800 troops, while India
contributed just above 5,500 troops. Except for Italy, all the top 20 contributors were countries
IN

in the Global South.

In news : India deployed a women-only platoon of peacekeepers to the UN Mission in


Abyei on the border of Sudan and South Sudan as part of the Indian Battalion in the United
Nations Interim Security Force, Abyei (UNISFA).

UN Peacekeeping Forces:

 UN peacekeepers work to prevent and resolve conflicts by monitoring


ceasefires, protecting civilians, supporting peace processes, and disarming
combatants.
 They also support elections, human rights, cultural heritage, and local security
forces. Peacekeepers operate with impartiality, ensuring consent from host
countries and warring parties.
 Despite challenges like volatile situations, complex political dynamics, resource
constraints, and potential attacks, they remain vital for conflict prevention and
mitigation.
 Over 120 countries contribute to UN peacekeeping, demonstrating the international
community's commitment to global peace and security.

85. Which of the following statements about UPI123Pay is/are correct?


1. Ministry of Finance has launched new UPI service for feature phones called

S
UPI123Pay.
2. Service needs internet connectivity to work

A
3. It is a three-step offline method to initiate and execute transactions that will work on
simple feature phones.
4I
Select the correct answer using the codes given below
a) 1 and 2 only
b) 2 and 3 only
IA
c) Only 3
d) Only 1

Answer: C
D

Explanation
IN

About UPI123Pay: Reserve Bank of India has launched new UPI service for feature
phones called UPI123Pay. It is a three-step offline method to initiate and execute
transactions that will work on simple feature phones. It has all features for
undertaking transactions except scan and pay. The service does not need an internet
connection for transactions.
 There are 4 options to make payments without internet connectivity:
Interactive Voice Response (IVR), Missed Call facility, Proximity sound-based
payment and App for feature phones.
 UPI 123Pay is largely an interactive voice response or IVR-based
technology, though it also has app-based functionality, proximity-sound
based payments and enabling payments through a missed call.
 QR code-based payment not possible in UPI 123Pay.
 Per transaction limit is 5000 and multiple bank accounts cannot be linked.

What is UPI Lite?


UPI Lite is a one-device wallet similar to those found on Paytm, Freecharge,
Mobikwik, and other popular digital payment apps. The functionality will enable you
to make near-real-time small-value payments using money stored in your wallet
without the need for an internet connection.

S
 Given that about 75 per cent of the total volume of retail transactions in
India (including cash transactions) are below ₹100 each and 50 per cent of

A
UPI transactions are for ₹200 each, UPI Lite will initially target
transactions of ₹200 or less.

4I
To begin with, the app will be designed only for debit transactions that is, to
send money from the app.
 The upper limit for payment is currently ₹200.
IA
 The key differentiating factor between a regular digital wallet and UPI Lite is
that an existing UPI customer need not repeat the KYC process to use UPI
Lite.
D

86. Which of the following ministries initiated the PRAJJWALA CHALLENGE?


a) Ministry of Rural Development
IN

b) Ministry of Women and Child Development


c) Ministry of Housing and Urban Affairs
d) Ministry of Skill Development

Answer: A

Explanation :
PRAJJWALA CHALLENGE
The Ministry of Rural Development launched it in an effort to get suggestions,
ideas, and activities that could revolutionize the rural economy. Individuals, social
enterprises, startups, the private sector, civil society, etc. are welcome to submit ideas
under a number of categories, with a particular emphasis on women and
marginalized communities, Regional Models & Economical remedies.

It is being introduced under the National Rural Livelihoods Mission (DAY-


NRLM) of the Deendayal Antyodaya Yojana. The goal of DAY-NRLM is to
encourage rural impoverished households to form SHGs and to offer them sustained
assistance so they can increase their earnings, diversify their sources of income, etc.

87. Agnipath Scheme in the Indian Armed Forces is a scheme wherein selected
candidates will be enrolled as Agniveers for period of how many years?.

S
a) Ten years
b) Four Years

A
c) Five years
d) Three Years

Answer: B
4I
IA
Explanation :

Agneepath scheme is a central government scheme launched in 2022. Under the


scheme, Indian youth can join the Indian Army for four years and will be able to serve
D

the country. This is a pan-India scheme so any Indian youth can apply for this job and
join the Indian Army.
IN

Name of the scheme Agneepath Yojana Entry Scheme, India

Beneficiary Youth of India

Purpose To Give Opportunities to the youth to serve in the armed forces as Agniveers.

Service Time 4 Years

Age Limit 18-25 years

Application process Online / Offline

Key features of the Agneepath scheme:


 It’s a Pan India married-based recruitment system. That means anyone from
anywhere in India can apply for this job.
 It’s a four-years tenure job.
 Opportunity to serve in the armed forces as Agniveers.
 Attractive monthly emoluments and handsome Seva Nidhi package.
 100% opportunity to apply for permanent enrollment.
 Enrollment model based on Agneepath scheme.
 Based on married organizational requirements, 25% of Agniveers selected based on a
Central, transparent, rigorous system, after 4 years.

Under this scheme, the youth will get an annual package of 4.76 lakh in the first year. 4th

S
year of their serving time the annual package will be increased to 6.9 to lakh.

A
After the 4 years of service, the service fund of rupees 11.7 lakh will be given to the youth.
This money will be totally tax-free. Apart from this other risk and hardship allowance will
4I
also be available. In total, this is a good package for youth to serve their four years in the army

88. In which of the following district South India's first DARK SKY RESERVE is located?
IA
a) Kodagu
b) Idukki
c) Shimoga
d) Ernakulam
D

Answer : A
IN

Explanation :

 Talacauvery in Kodagu District, Karnataka has emerged as South India's


Hanle. Hanle (in Ladakh) is India’s first Dark Sky Reserve.
 Dark Sky Reserve is a designation given to a public or private land with
exceptional or distinguished quality of starry nights, and nocturnal
environment that is specifically protected for its scientific, natural, educational,
cultural, heritage and/or public enjoyment.
 Dark Sky location depends on light pollution which is measured on Bortle
Scale.
 Bortle Scale measures night sky’s brightness on a nine-level numerical scale
with lesser numerical scale representing darkness.

89. What is the name of the committee formed by the Karnataka government to declare Tulu
as the second official language of Karnataka?

a) Mohan Alva
b) Shridhar Mohanty
c) Mahesh Hegde
d) K Sudhakar

S
Answer: A

A
Explanation:


4I
The Karnataka government has constituted a committee under educationist Mohan
Alva on declaring Tulu as Karnataka's second official language.
 The committee has been asked to study and submit a report to the government
IA
Recognising Tulu as a state language. Karnataka could give a major boost to the
demand to have it under the 8th Schedule, which lists official languages of India.
 If Tulu becomes the second official language of the state, it will be established as an
administrative language as well.
D

 About official language of state :


o Articles 346 and 347, the Legislature of a State may by law adopt any one or
IN

more of the languages in use in the State or Hindi as the language or


languages to be used for all or any of the official purposes of that State
o On a demand being made in that behalf the President may, if he is satisfied
that a substantial proportion of the population of a State desire the use of any
language spoken by them to be recognised by that State, direct that such
language shall also be officially recognised throughout that State or any
part thereof for such purpose as he may specify.

About Tulu: Tulu is a Dravidian language whose speakers are concentrated in


Dakshina Kannada and the southern part of Udupi of Karnataka in south-western India
and in the northern parts of the Kasaragod district of Kerala
90. Consider the following statement regarding RE-HAB Project and choose the correct
statement/s
1. The project has been launched by the Khadi and Village Industries Commission (KVIC).
2. This project is a sub-mission under the National Honey Mission

a) 1 only
b) 2 only
c) Both 1 and 2
d) Neither 1 nor 2

S
Answer: C

A
Explanation:

4I
A pilot project RE-HAB (Reducing Elephant-Human Attacks using Bees) has been
launched in Karnataka which entails installing bee boxes along the periphery of
the forest and the villages to mitigate human-elephant conflict. These spots are
IA
located on the periphery of Nagarahole National Park and Tiger Reserve, which is a
known conflict zone.

Implementing Agency: Project is an initiative of the Khadi and Village Industries


D

Commission (KVIC). It is a sub-mission of KVIC’s National Honey Mission.


IN

 Honey Mission: The KVIC has launched the Honey Mission to provide awareness,
training and ‘Bee Boxes’ along with Bee Colonies to the farmers. The mission was
launched in August 2017 in line with the ‘Sweet Revolution’. The 'Sweet Revolution'
was launched in 2016 to promote beekeeping and associated activities.

 Nagarhole National Park is also known as Rajiv Gandhi National Park. It was
established as a wildlife sanctuary in 1955 and was upgraded into a national park in
1988. It was declared as the 37th Tiger reserve under Project Tiger in 1999. The
Park lies in the Western Ghats and is a part of the Nilgiri Biosphere Reserve.
 Rivers: The Nagarahole River flows through the park, which joins the Kabini
River which also is a boundary between Nagarahole and Bandipur
National Park.

91. Consider the following statements with respect to NISAR Mission.


1. NISAR has been built by space agencies of the US and India under a partnership
agreement signed in 2014.
2. It is expected to be launched in ‘early 2024’ from Satish Dhawan Space Centre into a near-
polar orbit.
3. The satellite will operate for a minimum of three years.

S
How many of the above statements are correct?

A
a) Only one
b) Only two
c) Only three
4I
d) None of the above

Answer : C
IA
Explanation

NISAR Mission
D

 NISAR has been built by space agencies of the US and India under a partnership
agreement signed in 2014.
 It is expected to be launched in January 2024 from Satish Dhawan Space Centre
IN

into a near-polar orbit.


 The satellite will operate for a minimum of three years.
 It is a Low Earth Orbit (LEO) observatory. NISAR will map the entire globe in 12
days.

Key features of NISAR Mission€

 Using advanced radar imaging that will provide an unprecedented, detailed view of
Earth, the NASA-ISRO Synthetic Aperture Radar, or NISAR, satellite is
designed to observe and take measurements of some of the planet's most
complex processes.
 It is a 2,800 kilograms satellite consisting of both L-band and S-band Synthetic
Aperture Radar (SAR) instruments, which makes it a dual-frequency imaging radar
satellite.€
 While NASA has provided the L-band radar, GPS, a high-capacity solid-state
recorder to store data, and a payload data subsystem, ISRO (Indian Space Research
Organisation) has provided the S-band radar, the Geosynchronous Satellite Launch
Vehicle (GSLV) launch system and spacecraft.
 It has a 39-foot stationary antenna reflector, made of a gold-plated wire mesh; the
reflector will be used to focus “the radar signals emitted and received by the
upward-facing feed on the instrument structure.

S
92. ‘90-90-90 Target’ associated with which among the following disease ?

A
a) Tuberculosis
b) HIV
4I
c) Blood Pressure
d) Diabetes
IA
Answer : B

Explanation
D

90-90-90 Target

The idea behind the 90-90-90 target is to :


IN

 diagnose 90% of people who are HIV positive


 get 90% of the diagnosed HIV+ people on antiretroviral treatment
 90% of those on antiretroviral should be virally suppressed

93. Consider the following statements with respect to Mangrove Initiative for Shoreline
Habitats & Tangible Incomes (MISHTI).

1. MISHTI facilitates mangrove plantation along India’s coastline and on salt pan lands.
2. The programme will operate through convergence between MGNREGS, Campa Fund and
other sources.
3. MISHTI programme will aim at intensive afforestation of coastal mangrove forests
Choose the correct answer from the options given below.

a) 1 and 2 only
b) 2 and 3 only
c) 1 and 3 only
d) All of the above

Answer : D

Explanation

Mangrove Initiative for Shoreline Habitats & Tangible Incomes (MISHTI)

 MISHTI helps to facilitate mangrove plantation along India’s coastline and on salt

S
pan lands.
 The MISHTI programme will operate through convergence between MGNREGS,

A
Campa Fund and other sources.
 The MISHTI programme will aim at intensive afforestation of coastal mangrove


4I
forests.
India has such forests on both its Eastern and Western coasts with the Sundarbans
in Bengal being one of the largest mangrove forests on the planet.
IA
94. In which among the following location India’s Seed Vault (bank) is located ?
a) Kohima in Nagaland
b) Shimla in Himachal Pradesh
D

c) Chang La in Ladakh
d) Maddur in Karnataka
IN

Answer : C

Explanation

India’s seed bank :

 The Indian Seed Vault is a secure seed bank located in a high-altitude mountain
pass on the Chang La in Ladakh, India.
 It was built in 2010 jointly by the Defence Institute of High Altitude Research and
the National Bureau of Plant Genetic Resources, and is the second largest seed
bank in the world.
95. Consider the following statements with respect to Paris Club.

1. The Paris Club is a group of mostly western creditor countries that grew from a 1956
meeting in which Argentina agreed to meet its public creditors in Paris.
2. It describes itself as a forum where official creditors meet to solve payment difficulties
faced by debtor countries.
3. Paris Club objective is to find sustainable debt-relief solutions for countries that are unable
to repay their bilateral loans.

How many of the above statements are correct?

a) Only one

S
b) Only two
c) Only three

A
d) None of the above

Answer : C 4I
Explanation

Paris Club
IA
 The Paris Club is a group of mostly western creditor countries that grew from a
1956 meeting in which Argentina agreed to meet its public creditors in Paris.
 It describes itself as a forum where official creditors meet to solve payment
D

difficulties faced by debtor countries.


 Their objective is to find sustainable debt-relief solutions for countries that are
unable to repay their bilateral loans.
IN

96. The headquarter of ASEAN is located in which among the following country?

a) India
b) Indonesia
c) Thailand
d) Singapore

Answer : B

Explanation

ASEAN
 The Association of Southeast Asian Nations (ASEAN) is headquartered in Jakarta,
Indonesia
 ASEAN is an international organization made up of 11 countries :
 Indonesia, Vietnam, Laos, Brunei, Thailand, Myanmar, the Philippines, Cambodia,
Singapore, and Malaysia.
 The organization was established on August 8, 1967, with five member states:
Indonesia, Malaysia, the Philippines, Singapore, and Thailand.

S
A
4I
IA

97. The Karnataka Administrative Reforms Commission-2 (KARC-2) headed by?


D

a) T.M. Vijay Bhaskar


IN

b) R Ravikumar
c) Vandita Sharma
d) Shalini Rajnish

Answer : A

Explanation

 The Karnataka Administrative Reforms Commission-2 (KARC-2) headed by T.M.


Vijay Bhaskar, former Chief Secretary.
 KARC-2 recommended to the State government ban of private practice by
government doctors of all departments and modifying “rural quota” to “rural SSLC
board/government school quota” to enable more rural students to get seats in MBBS
course.

98. ‘HAL Helicopter Factory’ is located in which among the following district of Karnataka ?

a) Davanagere
b) Tumkur
c) Bidar
d) Bengaluru Urban

Answer : B

S
Explanation

A
Tumkur Helicopter Factory :

 The Tumkur Helicopter Factory is dedicated new greenfield facility is also Asia's


4I
largest helicopter manufacturing facility.
The project will enhance capacity and ecosystem to build more modern helicopters
in the categories like- Light Utility Helicopter, Light Combat Helicopters, Civil
IA
Advanced Light Helicopters, and Indian Multi-role Helicopter.
 This facility will enable India to meet its entire requirement of helicopters
indigenously.
 This unit will attain the distinction of enabling self-reliance in helicopter design,
D

development, and manufacture in India.


IN

99. Consider the following statements with respect to ‘Namma Clinics’.

1. Namma Clinics which will function as urban health and wellness centres.
2. Namma Clinics are aimed at providing comprehensive primary healthcare to the poor, the
vulnerable, and the underprivileged sections in urban areas.
3. Referral facilities will also be available and patients requiring higher treatment will be
referred to secondary and tertiary centres.

How many of the above statements are correct?

a) Only one
b) Only two
c) Only three
d) None of the above

Answer : C

Explanation

Namma Clinics

 The clinics, which will function as urban health and wellness centres, are aimed at
providing comprehensive primary healthcare to the poor, the vulnerable, and the
underprivileged sections in urban areas.
 Referral facilities will also be available and patients requiring higher treatment will
be referred to secondary and tertiary centres.

S
 Each clinic will have one doctor, one nursing staff, one lab technician, and one D
group worker.

A
 A total of 12 healthcare services packages will be available, including ante-natal,
post-natal care, adolescent care, immunisation, family planning, fertility,
4I
communicable diseases management, general and minor injuries, preventive care
for NCDs, oral care, eye care, ENT, mental health, geriatric care, and laboratory
services.
IA
 Tele-consultation, wellness activities like yoga and referral services will also be
available.

100. Which among the following was the slogan of 14th edition of Aero India 2023?
D

a) The Runway to a Billion Opportunities


b) Sky is the limit
IN

c) One Nation, One Show


d) Building the base for better future

Answer : A

Explanation

14th edition of Aero India 2023

 The first edition of Aero India was held in 1996.


 The 14th edition of Aero India 2023, was inaugurated by Prime Minister on February
13,2023.
 The slogan of 14th edition of Aero India 2023 was "The Runway to a Billion
Opportunities".

You might also like